U world Neurological all typed questions aspiration pneumonia.

Biochemistry
September 9, 2022
Improving Medication Adherence among Type II Home Healthcare Diabetic Patients
September 9, 2022

U world Neurological all typed questions aspiration pneumonia.

U world Neurological all typed questions

aspiration pneumonia.

The home health nurse teaches an elderly client with dysphagia some strategies to help limit repeated hospitalizations for aspiration pneumonia. Which statement indicates that the client needs further teaching?

· 1″| have to remember to raise my chin slightly upward when | swallow.” (56%)

 

 

Dysphagia increases the risk for aspiration of oropharyngeal secretions, gastric content, food, and/or fluid into the lungs. Aspiration of foreign material into the lungs increases the risk for developing aspiration pneumonia. Interventions to help decrease aspiration and resulting aspiration pneumonia in susceptible clients (eg, elderly, neurologic dysfunction, decreased cough or gag reflexes, decreased immunity, chronic disease), include the following:

· Swallowing 2 times before taking another bite of food. This clears food from the pharynx.

· Thickening liquids to assist swallowing

· Avoiding over-the-counter cold medications. Antihistamine cold preparation medications also have some anticholinergic properties, such as causing drowsiness, decreasing saliva (xerostomia) production, and making the mouth dry. Saliva is a lubricant, and it helps bind food together to facilitate swallowing.

· Sitting upright for at least 30-40 minutes after meals. This uses gravity to move food or fluid through the alimentary tract, decreases gastroesophageal reflux, and helps decrease risk for aspiration.

· Brushing teeth and using antiseptic mouthwash before and after meals. This reduces the bacterial count before eating because bacteria as well as food can be aspirated. After-meal use removes particles of food that can be aspirated later.

· Smoking cessation. Smoking decreases mucociliary clearance and increases bacterial count in the mouth.

 

(Option 1) Positioning the chin slightly downward toward the neck (chin-tuck) when swallowing can be effective in some clients with

dysphagia due to its facilitating closure of the epiglottis to help prevent tracheal aspiration.

Educational objective:

Teaching clients who are susceptible to aspiration about swallowing techniques, positioning, avoidance of over-the-counter cold

preparation medications (cause drowsiness and dry mouth), oral care, and smoking cessation can decrease the risk for aspiration

pneumonia

The nurse is caring for a client at risk for aspiration pneumonia due to a stroke. What nursing actions help prevent this potential

complication during hospitalization? Select all that apply.

· 1.Add a thickening agent to the fluids

· 2. Avoid administering sedating medications before meals

· 3. Place the client in an upright position during meals

· 5. Teach the client to flex the neck while swallowing

Aspiration pneumonia develops when aspirated material (eg, food, emesis, gastric reflux) causes an inflammatory response and

provides a medium for bacterial growth. At-risk conditions include cognitive changes (eg, dementia, head injury, stroke, sedation),

difficulty swallowing, compromised gag reflex, and tube feeding.

Aspiration-prevention measures include:

· Thicken liquids (eg, to nectar or honey consistency) for clients with dysphagia; thin liquids are more difficult to control whenswallowing (Option 1).

· Ensure that the client is fully awake before eating. The nurse should time the administration of sedating medications (eg, opioids, benzodiazepines) to avoid sedation during meals (Option 2).

· Elevate the head of the bed to 90 degrees during and for 30 minutes after meals, and never place the head of the bed lower than 30 degrees (Option 3).

· Encourage clients to facilitate swallowing by flexing the neck (chin to chest) (Option 5).

· Administer prescribed antiemetics (eg, ondansetron) as needed to prevent vomiting.

· Monitor for coughing, gagging, and pocketing food.

 

(Option 4) Performing strict handwashing and limiting sick visitors are important infection-control measures; however, they do not

prevent noninfectious aspiration pneumonia.

Educational objective:

Measures for preventing aspiration pneumonia include administering medications to prevent vomiting, avoiding mealtime sedation,

maintaining head-of-bed elevation at 30 degrees or more (90 degrees during and 30 minutes after meals), and encouraging neck

flexion while swallowing. Clients with dysphagia should receive thickened liquids and be monitored for coughing, gagging, and

pocketing food.

The client has increased intracranial pressure with cerebral edema, and mannitol is administered. Which assessment should the

nurse make to evaluate if a complication from the mannitol is occurring?

· 1. Auscultate breath sounds to assess for crackles (49%)

 

 

Mannitol (Osmitrol) is an osmotic diuretic used to treat cerebral edema (increased intracranial pressure) and acute glaucoma. When administered, mannitol causes an increase in plasma oncotic pressure (similar to excess glucose) that draws free water from the extravascular space into the intravascular space, creating a volume expansion. This fluid, along with the drug, is excreted through the kidneys, thereby reducing cerebral edema and intracranial pressure. However, if a higher dose of mannitol is given or it accumulates (as in kidney disease), fluid overload that may cause life-threatening pulmonary edema results. An early sensitive indicator of fluid overload is new onset of crackles auscultated in the lungs. To prevent these complications, clients require frequent monitoring of serum osmolarity, input and output, serum electrolytes, and kidney function.

(Option 2) Urine output would be expected to increase from the diuretic effect of mannitol. This is not a complication.

(Option 3) Glasgow Coma Scale scores range from 3-15. Improved mental status (orientation, alertness) is a desired effect of

treatment.

(Option 4) The presence of crackles is a more sensitive sign of fluid overload than pedal edema. Furthermore, in a bedridden client, the assessment should take place at a dependent part of the body, usually the sacral area.

Educational objective:

Mannitol is an osmotic diuretic used to treat cerebral edema and acute glaucoma. Normal kidney function and adequate urine output are crucial while administering this medication as mannitol accumulation can result in significant volume expansion, dilutional hyponatremia, and pulmonary edema.

Aclient is admitted to the hospital for severe headaches. The client has a history of increased intracranial pressure (ICP), which has required lumbar punctures to relieve the pressure by draining cerebrospinal fluid. The client suddenly vomits and states, “That’s weird, didn’t even feel nauseated.” Which action by the nurse is the most appropriate?

· 1. Document the amount of emesis (6%)

· 3. Notify the health care provider (HCP) (75%)

 

Unexpected and projectile vomiting without nausea can be a sign of increased ICP, especially in the client with a history of

increased ICP. The unexpected vomiting is related to pressure changes in the cranium. The vomiting can be associated with

headache and gets worse with lowered head position. The most appropriate action is to obtain a full set of vital signs and contact the HCP immediately.

(Option 1) Documentation is important, but it is not the priority action.

(Option 2) The head of the bed should be raised, not lowered, for clients with suspected increased ICP. Raising the head of the bed to 30 degrees helps to drain the cerebrospinal fluid via the valve system without lowering the cerebral blood pressure.

(Option 4) The vomiting is caused not by nausea but by pressure changes in the cranium. Anti-nausea medications are often not effective. Decreasing intracranial pressure will help the vomiting.

Educational objective:

Notify the HCP of signs/symptoms of increased ICP, including unexpected vomiting. The vomiting is often projectile, associated with headache, and gets worse with lowering the head position.

The nurse is assessing a client with advanced amyotrophic lateral sclerosis. Which of the following assessment findings does the

nurse expect? Select all that apply.

· 2. Difficulty breathing

· 3. Difficulty swallowing

· 4. Muscle weakness

 

Amyotrophic lateral sclerosis (ALS, Lou Gehrig disease) is a debilitating neurodegenerative disease with no cure. ALS causes progressive degeneration of motor neurons in the brain and spinal cord. Physical symptoms include fatigue, progressive muscle weakness, twitching and muscle spasms, difficulty swallowing, difficulty speaking, and respiratory failure (Options 2, 3, and 4). Most clients survive only 3-5 years after the diagnosis as there is no cure. Treatment focuses on symptom management. Interventions include:

 

· Respiratory support with noninvasive positive pressure (eg, bilevel positive airway pressure [BiPAP]) or invasive mechanical ventilation (eg, via tracheostomy)

· Feeding tube for enteral nutrition

· Medications to decrease symptoms (eg, spasms, uncontrolled secretions, dyspnea)

· Mobility assistive devices (eg, walker, wheelchair)

· Communication assistive devices (eg, alphabet boards, specialized computers)

(Option 1) Constipation due to decreased mobility is more common in ALS. Diarrhea is not seen.

(Option 5) Resting tremor is characteristic of parkinsonism,

Educational objective:

Amyotrophic lateral sclerosis causes motor neuron degeneration that leads to progressive muscle weakness, twitching and muscle spasms, difficulty swallowing, difficulty speaking, and respiratory failure. There is no cure. Treatment focuses on symptom management.

The daughter of an 80-year-old client recently diagnosed with Alzheimer disease (AD) says to the nurse, “| guess | can anticipate

getting this disease myself at some point.” What is an appropriate response by the nurse?

· 1. “Engaging in regular exercise decreases the risk of AD.” (52%)

 

 

The development of Alzheimer disease (AD) is related to a combination of genetic, lifestyle, and environmental factors. Clients with

AD are usually diagnosed at age 265. Early-onset AD is a rare form of the disease that develops before age 60 and is strongly related

to genetics. Children of clients with early-onset AD have a 50% chance of developing the disease.

For late-onset AD, the strongest known risk factor is advancing age. Having a first-degree relative (eg, parent, sibling) with late-

onset AD also increases the risk of developing AD (Option 2). Trauma to the brain has been associated with the development of AD

in the future. Brain trauma may be prevented by wearing seat belts and sports helmets and taking measures to prevent falls.

 

Research suggests that healthy lifestyle choices (eg, smoking cessation, avoiding excessive alcohol intake, exercising regularly,

participating in mentally challenging activities) reduce the risk for developing AD (Options 1 and 3).

 

(Option 4) Research has failed to confirm that exposure to aluminum products (eg, cans, cookware, antiperspirant deodorant) is

related to the development of AD.

Educational objective:

Research suggests that healthy lifestyle choices (eg, smoking cessation, avoiding excessive alcohol intake, exercising regularly,

participating in mentally challenging activities) reduce the risk for developing Alzheimer disease.

The emergency department nurse is assessing a client brought in after a car accident in which the client’s head hit the steering column. Which assessment findings would indicate that the triage nurse should apply spinal immobilization? Select all that apply.

· 1. Breath smells of alcohol

· 2. Client disoriented to place

· 5. Point tenderness over spine

 

Spinal immobilization is not a benign procedure. An acronym to help determine the need for spinal immobilization is NSAIDs:

N – Neurological examination. Focal deficits include numbness and decreased strength.

S – Significant traumatic mechanism of injury

A-Alertness. The client may be disoriented or have an altered level of consciousness (Option 2).

| – Intoxication. The client could have impaired decision-making ability or lack awareness of pain (Option 1).

D – Distracting injury. Another significant injury could distract the client from spinal pain.

S- Spinal examination. Point tenderness over the spine or neck pain on movement (if there is no midline tenderness) may be present

(Option 5).

(Option 3) The sensation of burning eyes could be related to many issues and does not necessarily have a direct correlation to spinal

trauma.

(Option 4) There is no direct correlation of multiple sclerosis (autoimmune progressive nerve demyelinization) with the need for spinal immobilization.

Educational objective:

Indications for spinal immobilization include abnormal neurological findings, significant mechanism of injury, change in orientation or level of consciousness, intoxication, distracting injury, and point tenderness over the spine.

The nurse is caring for a client who had a stroke two weeks ago and has moderate receptive aphasia. Which interventions should the

nurse include in the plan of care to help the client follow simple commands regarding activities of daily living (ADL)? Select all that

apply.

· 1. Ask simple questions that require “yes” or “no” answers

· 3. Remain calm and allow the client time to understand each instruction

· 4. Show the client pictures of ADL (eg, shower, toilet, and toothbrush) or use gestures

5. Speak slowly but loudly while looking directly at the client

Receptive aphasia refers to impairment or loss of language comprehension (ie, speech, reading) that is caused by a neurological

condition (eg, stroke, traumatic brain injury). The terms “aphasia” and “dysphasia” can be used interchangeably as both refer to

impaired communication; however, “aphasia” is more commonly used.

When assisting a client with receptive aphasia to complete activities of daily living, the nurse should avoid completing tasks for the

client and should instead encourage independence using appropriate communication techniques. Appropriate interventions to aid

communication include:

· Ask short, simple, “yes” or “no” questions (Option 1).

· Use gestures or pictures (eg, communication board) to demonstrate activities (Option 4).

· Remain patient and calm, allowing the client time to understand each instruction (Option 3).

(Option 2) Clients with aphasia often become frustrated due to inability to communicate effectively. Frustration does not result from

the nurse’s care, so reassigning the client to a different care provider is not an effective solution.

(Option 5) Eye contact is important in all communication, but raising the voice will not help. Speaking loudly will not improve

comprehension and may increase anxiety and confusion.

Educational objective:

Receptive aphasia refers to impairment or loss of language comprehension. Appropriate interventions to aid communication include

asking short, simple, “yes” or “no” questions; using hand gestures or pictures to demonstrate activities; and patiently allowing the

client time to understand each instruction.

The graduate nurse cares for several poststroke clients. Which of the following nursing interventions are appropriate? Select all that

apply.

· 1. Implement fall precautions for the client with cerebellar stroke

· 3. Initiate swallow precautions for the client with cranial nerves |X and X affected

· 4. Place spoon within field of vision for the client with homonymous hemianopsia

 

Strokes cause different neurological deficits depending on the location of the affected area within the brain and the extent of injury. Cerebellar deficits affect balance and equilibrium; fall precautions are appropriate (Option 1). Cranial nerves IX (glossopharyngeal) and X (vagus) control the gag and swallowing mechanisms, making swallow precautions necessary (Option 3). Blindness in the same half of each visual field, homonymous hemianopsia, is suspected when clients ignore objects on one side. Initially, the nurse assists (eg, places utensil in unaffected visual field), but the client must learn to turn the head to scan the environment (Option 4).

(Option 2) Astroke affecting cranial nerve VII, the facial nerve, can cause an asymmetrical smile or inability to raise one eyebrow. Increased light is unnecessary as vision is not affected.

(Option 5) Clients experiencing receptive aphasia, impaired comprehension of speech and writing, typically have injury to the

Wernicke area of the brain, located in the left temporal lobe. The nurse would not speak louder as this does not aid comprehension. The nurse should speak clearly, ask “yes” or “no” questions, and use gestures and pictures to increase understanding.

Educational objective:

Strokes cause different neurological deficits depending on the location and extent of injury. Cerebellar deficits affect balance and require fall precautions, cranial nerve IX and X injuries can impair swallowing, and a client with homonymous hemianopsia will not see objects on the affected side.

Aclient is admitted to the hospital with an exacerbation of myasthenia gravis. What are the appropriate nursing actions? Select all that apply.

· 1. Administer an anticholinesterase drug AC

· 2. Anticipate a need for an anticholinergic drug

· 4. Encourage semi-solid food consumption

· 5. Teach the necessity for annual flu vaccination

Myasthenia gravis is an autoimmune disease involving a decreased number of acetylcholine receptors at the neuromuscular

junction. As a result, there is fluctuating weakness of skeletal muscles, most often presented as ptosis/diplopia, bulbar signs

(difficulty speaking or swallowing), and difficulty breathing. Muscles are stronger in the morning and become weaker with the

day’s activity as the supply of available acetylcholine is depleted.

Treatment includes anticholinesterase drugs (pyridostigmine [Mestinon]) that are administered before meals so that the client’s

ability to swallow is strongest during the meal (Option 1).

Semi-solid foods (easily-chewed foods) are preferred over solid foods (to avoid stressing muscles involved in chewing and

swallowing) or liquids (aspiration risk) (Option 4).

 

All clients with a serious chronic co-morbidity should receive the annual flu vaccine (also the pneumonia vaccine if appropriate) as

they are more likely to have a negative outcome if the illness is contracted. It is especially important in clients with myasthenia gravis

as the flu (or pneumonia) would tax the already compromised respiratory muscles (Option 5).

 

(Option 2) An anticholinergic drug, such as atropine, is used for treatment in a cholinergic crisis (eg, the medication is too high or

there is excess acetylcholine). The need would not be anticipated during a myasthenic crisis (eg, exacerbation of myasthenia gravis),

which is usually a result of too little medication related to noncompliance, illness, or surgery.

 

(Option 3) The skeletal muscles are involved in myasthenia gravis; dysfunction of the reflexes or central nervous system affects

bowel and bladder control. This issue is classic with multiple sclerosis.

Educational objective:

Myasthenia gravis involves reduction of acetylcholine receptors in the skeletal muscles; this decreases the strength of muscles used

for eye and eyelid movements, speaking, swallowing, and breathing. Treatment includes administration of anticholinesterase drugs

before meals, easily-chewed foods, and appropriate vaccinations.

 

The client comes to the emergency department status post fall. The client is squinting both eyes and reports sudden blurry vision.

The nurse is aware that this deficit reflects injury to which area of the brain? Left-clicking the mouse will put an X to show the

answer before submitting the question.

 

The occipital lobe of the brain registers visual images. Injury to the occipital lobe could result in a deficit with vision. The nurse

should notify the health care provider immediately and document the finding.

The frontal lobe controls higher-order processing, such as executive function and personality. Injury to the frontal lobe often results in behavioral changes.

The temporal lobe integrates visual and auditory input and past experiences.

The parietal lobe integrates somatic and sensory input.

Educational objective:

The occipital lobe receives visual images. The frontal lobe controls executive function and personality. The temporal lobe receives

auditory input. The parietal lobe receives sensory input.

The nurse admits a client who fell off a 20-ft (6-m) ladder. On arrival in the emergency department, the client is arousable but

lethargic. What is the nurse’s priority action?

· 3. Obtain a Glasgow Coma Scale score (68%)

 

After trauma to a client (eg, fall), the nurse performs an emergency or trauma assessment that includes a primary and secondary

survey (assessment). The primary assessment determines the status of the airway, breathing, and circulation (ABCs). Next, the

nurse evaluates disability (D) of neurological function using the Glasgow Coma Scale (GCS).

 

The GCS measures the client’s level of consciousness by assessing the best eye opening response, best verbal response, and best

motor response. The lower the GCS score, the higher the risk for the client to develop complications (eg, loss of airway patency,

increased intracranial pressure).

 

(Options 1, 2, and 4) Although a health history, head-to-toe assessment, and notation of the client’s level of pain are essential for the

overall assessment, they are considered part of the secondary survey. This survey’s purpose is to get a complete picture of the

injuries, but only after the client’s priority needs have been addressed.

 

Educational objective:

After trauma to a client (eg, fall), the nurse performs a primary survey to determine status of airway, breathing, circulation, disability

(eg, Glasgow Coma Scale to assess neurological impairment), and exposure. Health history, head-to-toe assessment, and level of pain are part of the secondary survey.

 

The nurse is caring for an assigned team of clients. Which client is the priority for the nurse at this time?

· 3. Client with epilepsy puts on call light and reports having an aura (78%)

 

An aura is a sensory perception that occurs prior to a complex or generalized seizure. The client will most likely have a tonic-clonic

seizure soon, and the nurse should attend to this client first to ensure safety measures (ie, seizure precautions) are in place.

 

(Option 1) Guillain-Barré syndrome is an ascending symmetrical paralysis. It can move upward rapidly or relatively slowly (over

days/weeks). Respiratory compromise is the worst complication. A client with paralysis at the level of the knee after 24 hours would

not take priority over a client who will have a seizure in few minutes.

 

(Option 2) Scanning speech is a dysarthria in which there are noticeable pauses between syllables and/or emphasis on unusual

syllables. It is an expected finding with multiple sclerosis.

 

(Option 4) Fibromyalgia involves neuroendocrine/neurotransmitter dysregulation. Clients experience widespread pain with point

tenderness at multiple sites, including the neck and shoulders. This client is not a priority.

 

Educational objective:

An aura is a sensory warning that a complex or generalized seizure will occur. It is a priority over stable or expected findings such as

point tenderness in fibromyalgia, low-level location of paralysis in Guillain-Barré syndrome, or scanning speech in multiple sclerosis.

The nurse in the outpatient clinic is speaking with a client diagnosed with cerebral arteriovenous malformation. Which statement

would be a priority for the nurse to report to the health care provider?

 

· 4. “| took an acetaminophen in the waiting room for this bad headache.” (51%)

 

An arteriovenous malformation (AVM) is a tangle of veins and arteries that is believed to form during embryonic development.

The tangled vessels do not have a capillary bed, causing them to become weak and dilated. AVMs are usually found in the brain and

can cause seizures, headaches, and neurologic deficits.

 

Treatment depends on the location of the AVM, but blood pressure control is crucial. Clients with AVMs are at high risk for having

an intracranial bleed as the veins can easily rupture because they lack a muscular layer around their lumen. Any neurologic

changes, sudden severe headache, nausea, and vomiting should be evaluated immediately as these are usually the first symptoms

of a hemorrhage (Option 4).

 

(Option 1) The report of dyspnea may prompt further evaluation depending on the type of exercise performed, but it is not the

 

priority. Clients with AVMs should be discouraged from engaging in heavy exercise as it increases blood pressure.

 

(Option 2) Clients with AVMs should avoid smoking to prevent hypertension. This client needs education on smoking cessation, but

it is not the priority.

 

(Option 3) Reports of not feeling well and sleeping a lot may be related to the headache and possible hemorrhage, but this alone

would not prompt a call to the health care provider.

 

Educational objective:

An arteriovenous malformation is a congenital deformity of tangled blood vessels often occurring in the brain. These vessels may

weaken and rupture, causing an intracranial hemorrhage. Any neurologic changes and severe headache need to be addressed

immediately as these may indicate hemorrhage.

 

The nurse is caring for a client in the medical-surgical unit who has delirium according to the Confusion Assessment Method

assessment tool. Which of the following assessment findings are likely contributing to the client’s delirium? Select all that apply.

· Multiple doses of IV hydromorphone administered in the past 12 hours

· 2. Serum sodium of 123 mEq/L (123 mmol/L)

· 3. SpO, of 82% on room air

· 4. Temperature of 103.1 F (39.5 C)

· 5. Urine culture positive for gram-positive cocci in chains

Delirium is characterized by an acute or fluctuating change in mental status that is often reversible and related to an underlying

medical condition. Evidence-based assessment tools, such as the Confusion Assessment Method (CAM), help clinicians quickly

recognize delirium. Criteria of the CAM tool include an acute or fluctuating change in mentation, inattention, disorganized thinking,

and altered level of consciousness.

Precipitating factors of delirium are numerous and include:

· Medications (eg, opioids, anticholinergics) (Option 1)

· Electrolyte imbalances (eg, hyponatremia) (Option 2)

· Hypoxia (Option 3)

· Acute infection (eg, fever, positive culture) (Options 4 and 5)

· Sleep deprivation

· Dehydration or malnutrition

· Metabolic disorders (eg, hypoglycemia)

 

Nursing interventions include treating the underlying cause as prescribed to resolve delirium (eg, antibiotics, supplemental oxygen),

maintaining a safe environment (eg, continuous monitoring, room near the nurses’ station, bed alarm), reorienting the client frequently,

promoting a regular sleep cycle, providing familiar items from home, and encouraging family and friends to stay with the client.

 

Educational objective:

The Confusion Assessment Method is an assessment tool that helps clinicians quickly recognize delirium, which is characterized by

an acute or fluctuating change in mental status and is often caused by medications (eg, opioids) or an underlying medical condition

(eg, hypoxia, electrolyte imbalances, infection).

 

Aspeeding driver sustained a closed-head injury in an acceleration/deceleration accident from striking a tree front end first. Based on

the coup-contrecoup phenomenon, which assessments are most likely to be affected related to the involved areas of the brain?

· 1. Expressive speech, vision (72%)

 

Coup-contrecoup injury occurs when a body in motion stops suddenly (eg, head hits car windshield), causing contusions (bruising) of

brain tissue as the brain moves back and forth within the skull. First, the soft tissue strikes the hard skull in the same direction as the

momentum (coup). As the body bounces back, the brain strikes the opposing side of the skull (contrecoup).

 

When the forward collision occurred, the frontal lobe most likely suffered the primary impact (coup). Executive function, memory,

speech (Broca area), and voluntary movement are controlled by the frontal lobe. The contrecoup most likely injured the occipital

lobe, where vision is processed.

 

(Option 2) The temporal lobe (lateral aspect of the brain) controls hearing and integrates sensory data (eg, auditory, visual, somatic).

The Wernicke speech area in the temporal lobe is responsible for language comprehension. Light touch is processed by the sensory

cortex in the parietal lobe.

 

(Option 3) An interruption of sensory function indicates injury to either the spinal column or the parietal lobe. These injuries affect

proprioception (awareness of body positioning) and graphesthesia (ability to identify writing on the skin, by touch).

 

(Option 4) The Weber test screens for conductive hearing loss by checking whether a tuning fork held along the midline of the head

is heard evenly in both ears. Cranial nerve | is the olfactory nerve. Hearing and smell are both processed by the temporal lobe.

 

Educational objective:

Coup-contrecoup injuries usually affect the frontal and occipital lobes. The frontal lobe controls executive function, memory, speech,

and motor skills. The occipital lobe processes vision.

Aclient with stroke symptoms has a blood pressure of 240/124 mm Hg. The nurse prepares the prescribed nicardipine intravenous

(IV) infusion solution correctly to yield 0.1 mg/mL. The nurse then administers the initial prescription to infuse at 5 mg/hr by setting the infusion pump at 50 mL/hr. What is the nurse’s priority action at this time?

 

· 2. Increase pump setting to correct administration rate to 100 mL/hr (8%)

· 3. Keep systolic blood pressure above 170 mm Hg (35%)

· 4. Monitor for a widening QT interval (36%)

 

A client with an acute stroke presentation (brain attack) requires “permissive hypertension” during the first 24-48 hours to allow for

adequate perfusion through the damaged cerebral tissues. However, the blood-brain barrier is no longer intact once the blood

pressure is >220/120 mm Hg. Therefore, “mild” lowering is required, usually to a systolic pressure that is not below 170 mm Hg.

 

Nicardipine (Cardene) is a prototype of nifedipine and is a potent calcium channel blocking vasodilator. It takes effect within 1

minute of IV administration. It is essential to monitor that the blood pressure is not being lowered too quickly or too slowly as this

would extend the stroke. Hypotension can occur with or without reflex tachycardia. The drug must be discontinued if hypotension

or reflex tachycardia occurs.

 

(Option 1) It is important to watch for signs of adequate perfusion to the kidneys as indicated by adequate urine output.

Approximately 25% of the cardiac output goes to the kidney. However, the priority is to monitor for the rapid onset of effect that this

potent antihypertensive agent has after initiation. The nurse should not wait until the urine output drops.

 

(Option 2) The initial rate set by the nurse is correct as desired dose/dose on hand is 5 mg/hr divided by 0.1 mg/mL and equals 50

mL/hr.

 

(Option 4) Widening of the QT interval can increase the risk of life-threatening torsades de pointes. It is most commonly seen with

haloperidol (Haldol), methadone, ziprasidone (Geodon), and erythromycin. However, this is not an expected complication of

nicardipine.

 

Educational objective:

Nicardipine (Cardene) is a potent calcium channel blocking vasodilator. The priority nursing intervention with IV administration is to

monitor the hypotensive effects of this drug.

 

The nurse provides education for caregivers of a client with Alzheimer disease. Which instructions should the nurse include? Select

all that apply.

 

 

· 2. Decrease the client’s anxiety by limiting the number of choices offered

· 3. Redirect the client if agitated by asking for help with a task or going for a walk

· 4. Remember to interact with the client as an adult, regardless of childlike affect

 

Strategies for caring for clients with Alzheimer disease address progressive memory loss and declining ability to communicate, think

clearly, and perform activities of daily living. Caregivers should also learn to manage clients’ problematic behavior and mood swings.

Therapeutic guidelines include:

· Use distraction and redirection (eg, going for a walk) to manage agitation (Option 3).

· Speak slowly and use simple words and yes-or-no questions.

· Do not try to rationalize with the client.

· Use visual cues when giving directions.

· Interact with the client as an adult, even as the client regresses to childlike affect and behavior; respect client dignity by avoiding use of pet names (eg, “honey,” “sweetie,” “darling”) (Option 4).

· Break down complex activities into steps with simple instructions.

· Decrease the client’s anxiety by limiting the number of choices (Option 2).

 

(Option 1) Allow plenty of time for task completion. The client cannot process information rapidly, and hurrying or rushing can cause

agitation or anxiety.

 

(Option 5) Ask questions that can be answered with yes, no, or very few words. Do not ask open-ended questions, which can

overwhelm the client and cause increased stress and frustration.

 

Educational objective:

Caregivers for clients with Alzheimer disease should communicate with the client using yes-or-no questions and simple, step-by-step

instructions; treat the client as an adult; limit the number of choices; and allow plenty of time for task completion. Agitated clients can

be redirected with new activities (eg, going for a walk).

 

The clinic nurse is assessing a previously healthy 60-year-old client when the client says, “My hand has been shaking when | try to cut

food. | did some research online. Could | have Parkinson’s disease?” Which response from the nurse is the most helpful?

 

· 4 “Tell me more about your symptoms. When did they start?” (97%)

 

Parkinson’s disease (PD) is a chronic, progressive neurodegenerative disorder that involves degeneration of the dopamine-

producing neurons. Damage to dopamine neurons makes it difficult to control muscles through smooth movement. PD is

characterized by a delay in initiation of movement (bradykinesia), increased muscle tone (rigidity), resting tremor, and shuffling

gait.

 

The most helpful response by the nurse is the one that acknowledges the concern of the client and also asks for more information.

The nurse should assess for additional information and perform a more focused physical assessment given this new information

(Option 4).

 

(Option 1) It is incorrect to say that the client is too young to have PD although it is usually seen after age 60; about 15% of PD cases

are diagnosed before age 50.

 

(Option 2) Although the nurse should encourage the client and family to discuss concerns with the health care provider, this is not the

most helpful response.

 

(Option 3) Although the typical parkinsonian tremor occurs at rest and not during purposeful movement, it is not helpful to dismiss a

concern without probing for more information.

 

Educational objective:

Therapeutic communication includes acknowledging concerns and probing for additional information as part of an assessment.

Aclient was struck on the head by a baseball bat during a robbery attempt. The nurse gives this report to the oncoming nurse at shift

 

change and conveys that the client’s current Glasgow Coma Scale (GCS) score is a'”10.” Which client assessment is most important

for the reporting nurse to include?

 

· 2. GCS score was “11” one hour ago (67%)

 

The GCS quantifies the level of consciousness in a client with acute brain injury by measuring eye opening (alertness), verbal

response (orientation), and motor response (eg, obeying a command, frontal lobe function). The maximum score on the GCS is 15

and the lowest is 3. If a client is trending for deterioration, this should always be noted in neurological assessments. A numerical

decline of a single number in 1 hour is significant. A criticism of the GCS score is that it is not that precise.

 

(Option 1) Orientation to place is part of the GCS score (under best verbal). The total score and the negative trend are more

indicative of the client’s condition than any individual GCS component.

 

(Option 3) This client’s vital signs are within normal limits and are not significant. It would be more important to communicate if there

is absence of Cushing’s triad (bradycardia, bradypnea/Cheyne-Stokes, and widening pulse pressure) or to give a brief summary of

overall vital signs (eg, “normal”); exact readings are accessible and can be recited if the oncoming nurse needs to know them.

However, reporting the negative neurological trend in the GCS score is the priority.

 

(Option 4) Although it is important to be aware of allergies, the oncoming nurse can find that information on the chart if these

medications are ordered. The main concern is blunt head trauma and not infection; therefore, it is unlikely that the oncoming nurse

will be required to know this information and need to administer antibiotics.

 

Educational objective:

It is a priority to report a negative neurological trend as evidenced by GCS score in a client with blunt head trauma.

The emergency department nurse receives several prescriptions for a client who was found unresponsive after drinking beer and

consuming unidentified pills. Which prescription should the nurse implement first? Click on the exhibit button for additional

information.

 

· 1. Administer IV push naloxone once now (75%)

X 4. Obtain urine sample for drug abuse screening ASAP (2%)

The goals of emergency care for the client with suspected substance abuse who exhibits signs of central nervous system

depression (eg, altered level of consciousness, bradypnea, hypotension, bradycardia) are to promote adequate ventilation and

oxygenation and preserve hemodynamic stability. Interventions are prioritized according to the ABCs (ie, airway, breathing,

circulation). Initial actions involve maintaining patency of the client’s airway, including appropriate positioning, oropharyngeal

suctioning, and artificial airway placement (if needed).

 

Respiratory depression occurring after the ingestion of an unknown substance (eg, depressants [opioids, benzodiazepines,

barbiturates]) should initially be treated with administration of reversal agents (eg, naloxone, flumazenil). Naloxone rapidly reverses

the effects of opioids and may restore spontaneous respiration and normal ventilatory pattern, averting initiation of mechanical

ventilation, the possibility of respiratory arrest, and death (Option 1).

 

(Options 2 and 4) Obtaining blood and urine for toxicology screening assists in guiding care decisions but should occur after

 

interventions that support the client’s airway, breathing, and circulation.

 

(Option 3) Administration of IV fluids to support blood pressure and prevent dehydration should be performed after securing the

client’s airway and supporting effective breathing.

 

Educational objective:

Nurses providing emergency care to clients with suspected substance abuse who exhibit signs of central nervous system depression

(eg, bradypnea, bradycardia) prioritize interventions according to the ABCs (ie, airway, breathing, circulation). Administration of

naloxone is a priority action in the setting of respiratory depression from an unknown substance because it rapidly reverses the

depressant effects of opioids.

 

The clinic nurse educator is developing a teaching plan for the following 6 clients. The nurse should instruct which client to avoid the

Valsalva maneuver when defecating? Select all that apply.

· 1. 22-year-old man with a head injury sustained during a college football game

· 3. 56-year-old man 2 weeks post myocardial infarction

· 5. 74-year-old man with portal hypertension related to alcohol-induced cirrhosis

· 6. 82-year-old woman 1 week post cataract surgery

 

The Valsalva maneuver (straining during defecation) involves holding the breath while bearing down on the perineum to pass a stool.

Straining to have a bowel movement is to be avoided in clients recently diagnosed with increased intracranial pressure, stroke, or

head injury as straining increases intra-abdominal and intrathoracic pressure, which raises the intracranial pressure (Option 1).

 

The vagus nerve is stimulated when bearing down; this temporarily slows the heart and decreases cardiac output, leading to potential

cardiac complications in clients with heart disease (Option 3).

 

Straining increases intra-abdominal and intrathoracic pressure and should be avoided in clients diagnosed with portal hypertension

 

related to cirrhosis due to the risk of variceal bleeding (Option 5).

The maneuver increases intraocular pressure and is contraindicated in clients with glaucoma and recent eye surgery (Option 6).

 

(Option 2) The otherwise healthy client recovering from reconstructive augmentation mammoplasty is not at risk for complications

related to the Valsalva maneuver.

 

(Option 4) The client recently diagnosed with pancreatic cancer is not at risk for complications related to the Valsalva maneuver.

 

Educational objective:

The Valsalva maneuver is contraindicated in the client diagnosed with increased intracranial pressure, stroke, head injury, heart

disease, glaucoma, eye surgery, abdominal surgery, and liver cirrhosis.

The nurse is caring for a client with increased intracranial pressure (ICP). Which statement by the unlicensed assistive personnel

would require immediate intervention by the nurse?

 

· 4. “You should do deep breathing and coughing exercises.” (81%)

 

Clients with elevated ICP should avoid anything that increases intrathoracic or intraabdominal pressure as these also indirectly

increase ICP. These activities include straining, coughing, and blowing the nose. Respiratory interventions, if needed, may include

deep breathing and incentive spirometry in the absence of coughing.

 

The head of the bed should be maintained at 30 degrees, high enough to allow for cerebrospinal fluid drainage, but low enough to

maintain cerebral perfusion pressure. Clients should have minimal stimuli, including no bright lights or multiple visitors, as stimulation

can increase ICP.

 

(Option 1) The head of the bed should be raised to 30 degrees and maintained. This may require follow-up, but not immediate

intervention, by the nurse.

 

(Option 2) Turning down the lights is appropriate as clients with ICP benefit from a quiet, nonstimulating environment.

(Option 3) Bringing items closer to the client prevents straining from reaching and is appropriate.

 

Educational objective:

Clients with increased ICP should be encouraged not to cough, strain, or increase abdominal or thoracic pressure. The head of the

bed should be maintained at 30 degrees, and stimulation in the room should be minimized.

 

The nurse is caring for a client after a lumbar puncture (spinal tap). Which client assessment is most concerning and requires a

nursing response?

 

· 2. Insertion site dressing saturated with clear fluid (74%)

 

A lumbar puncture involves removing a sample of cerebrospinal fluid through a needle inserted between vertebrae. Elevated

intracranial pressure is a contraindication to performing a lumbar puncture. The client is placed in the fetal position or sitting and

leaning over a table. Continued leaking fluid indicates that the site did not seal off and a blood patch (autologous blood into the

epidural space) is required.

 

(Option 1) Fluids are encouraged to help replace the cerebrospinal fluid.

(Option 3) The client should lie flat for at least 4 hours. The prone or supine position is recommended to help prevent a headache.

 

(Option 4) Up to 5%-30% of clients have the common complication of headache. It is thought to be a result of leakage of fluid

through the dural puncture site. The symptom is treated and is normally self-limiting.

 

Educational objective:

After a lumbar puncture, cerebrospinal fluid leakage from the puncture site requires health care provider notification for a blood patch.

A headache after the procedure is an expected finding. The client should lie flat and increase fluid intake afterwards.

The nurse receives the assigned clients for today on a neurology unit. The nurse should check on which client first?

 

· 3. Client with history of T2 spinal injury who has diaphoresis, pulse 54/min, and hypertension (91%)

 

Autonomic dysreflexia (autonomic hyperreflexia) is a massive, uncompensated cardiovascular reaction by the sympathetic nervous

system (SNS) in a spinal injury at T6 or higher. Due to the injury, the parasympathetic nervous system cannot counteract the SNS

stimulation below the injury. Classic triggers are distended bladder or rectum. Classic manifestations include severe hypertension,

throbbing headache, marked diaphoresis above the level of injury, bradycardia, piloerection (goose bumps), and flushing. This is an

emergency condition requiring immediate intervention. Management includes raising the head of the bed and then treating the cause.

 

(Option 1) The Glasgow Coma Scale (GCS) is an objective scale used to monitor alertness/mental functioning in an acute head

injury. The best score is 15, and the worst is 3. This client has an improving trend and is not a priority.

 

(Option 2) Myasthenia gravis is an autoimmune disease manifesting mainly as muscle weakness and ptosis. The muscle weakness

increases with activity, and by the end of the day, ptosis is present. These are expected findings for this condition, and so this client is

not a priority. However, clients with myasthenic crisis can have respiratory failure, which, if it occurs, would be a priority.

 

(Option 4) Transverse myelitis (spinal cord inflammation) usually results from a recent viral infection. Classic symptoms include

paralysis, urinary retention, and bowel incontinence. Some clients recover, but many have permanent disability. Normal muscle

strength is 5 on a scale of 0-5. Weakened muscle strength (2+ means only able to move laterally, not able to lift up against gravity)

would be an expected finding.

 

Educational objective:

Autonomic dysreflexia in a client with a spinal cord injury is a priority and requires emergency intervention. Classic triggers are

distended bladder or rectum. Management includes raising the head of the bed and then treating the cause (eg, Foley catheter kinks).

 

The nurse is caring for a client with a history of headaches. The client has talked to the nurse, smiled at guests, and maintained

stable vital signs. The nurse notes the following changes in the client’s status. Which assessment finding is critical to report to the

health care provider (HCP)?

 

 

· 2. Flat affect and drowsiness (82%)

The level of consciousness is the most important, sensitive, and reliable indicator of the client’s neurological status. Changes in the

level of consciousness can represent increased intracranial pressure and reduced cerebral blood flow.

Changes in vital signs usually do not appear until intracranial pressure has been elevated for some time, or they may be sudden in

cases of head trauma.

(Option 1) The blood pressure is slightly elevated but does not warrant immediate action or signify an emergency situation.

(Option 3) A poor appetite is not an emergency finding or situation.

 

(Option 4) The respiratory rate is slightly low, but if it is not irregular it is not an emergency as a single observation. This finding

would warrant further assessment and continued monitoring, but it is not as significant as the change in level of consciousness.

 

Educational objective:

Achange in level of consciousness for the neurological client should be reported to the HCP. The level of consciousness is the most

 

sensitive and reliable indicator of the client’s neurological status.

The nurse educates the caregiver of a client with Alzheimer disease about maintaining the client’s safety. Current symptoms include

occasional confusion and wandering. Which of the following responses by the caregiver show correct understanding? Select all that

apply.

 

· 1. “Grab bars should be installed in the shower and beside the toilet.”

· 2. “| will place a safe return bracelet on the client’s wrist.”

· 3. “Keyed deadbolts should be placed on all exterior doors.”

· 5. “Throw rugs and clutter will be removed from the floors.”

 

Alzheimer disease (AD) is a form of dementia that causes progressive decline of cognitive and physical abilities. The nurse should

educate the client/caregiver to prepare for current and future safety needs. Interventions evolve to meet client needs at each stage

of disease progression. Safety promotion for the client with moderate AD includes:

 

· Keyed deadbolts (with keys removed) and close supervision to provide a controlled environment for wandering (Option 3)

· Medical identification/location devices (eg, bracelets, shoe inserts) in case the client wanders outside the designated area (Option 2)

· Decreased water heater temperature and “hot” and “cold” labels on faucets to prevent burns

· Household hazards (eg, gas appliances, rugs, toxic chemicals) removed to prevent injury (Option 5)

· Grab bars installed in showers and tubs (Option 1)

 

(Option 4) All medications should be out of the client’s reach or locked away. A confused person may not remember the day of the

week and take more or less medication than prescribed.

 

Educational objective:

For clients with moderate Alzheimer disease, caregivers should provide a controlled environment for safe wandering (eg, throw rugs

and clutter removed, exterior doors secured), and the client should wear an identification/location device (eg, bracelet). All

medications should be out of reach or locked away. Hazards (eg, gas appliances, rugs, toxic chemicals) should be removed. Grab

bars should be installed in showers and tubs.

 

The emergency department nurse is triaging clients. Which neurologic presentation is most concerning for a serious etiology and

should be given priority for definitive treatment?

 

· 4. Temple region hit by ball, loss of consciousness, but Glasgow Coma Scale score is now 14 (41%)

Epidural hematoma is an accumulation of blood between the skull bone and dura mater. The majority of epidural hematomas are

associated with fracture of the temporal bone and subsequent rupture or tear of the middle meningeal artery. The bleed is

 

arterial in origin, and so hematoma develops quickly.

 

The clinical presentation of epidural hematoma is characteristic. The client may lose consciousness at the time of impact. The client

then regains consciousness quickly and feels well for some time after the injury. This transient period of well-being is called a lucid

interval. It is followed by a quick decline in mental function that can progress into coma and death.

 

(Option 1) Bell’s palsy (peripheral facial paralysis) is an inflammation of the facial nerve (CN VII) in the absence of other disease

etiologies, such as stroke. There is flaccidity of the affected side with drooling. This differs from the concerning drooling with epiglottis in which the client’s throat is too sore and/or swollen to swallow saliva. Treatment includes steroids, measures to relieve symptoms, and protection of the eye (which may not close tightly), but the condition is not emergent.

 

(Option 2) Multiple sclerosis is a chronic, relapsing, and remitting degenerative disorder involving the brain, optic nerve, and spinal

cord, Optic neuritis is a common presentation but is not life-threatening.

 

(Option 3) Trigeminal neuralgia (tic douloureux) presents with paroxysms of unilateral excruciating facial pain along the distribution of the trigeminal nerve (CN \V) that are often triggered by touch, talking, or hot/cold air or intake. Carbamazepine (Tegretol) is the drug of choice; the condition is not life-threatening.

 

Educational objective:

The classic presentation of intracranial epidural bleed is loss of consciousness to a period of lucidity and then gradual loss of

consciousness. The bleed is arterial in origin, and so hematoma develops quickly. Emergent diagnosis and treatment are needed to

prevent brain stem herniation. The nurse receives report for 4 clients in the emergency department. Which client should be seen first?

 

· 4. 70-year-old with atrial fibrillation and a closed-head injury waiting for brain imaging who reports a headache and had emesis of 200 mL (41%)

 

Aclient with a neurological injury (eg, head trauma, stroke) is at risk for cerebral edema and increased intracranial pressure (ICP), a

life-threatening situation. The client with atrial fibrillation may also be taking anticoagulants (eg, warfarin, rivaroxaban, apixaban,

dabigatran), making a life-threatening intracranial bleed even more dangerous. The nurse should perform a neurologic assessment

(eg, level of consciousness, pupil response, vital signs) immediately.

 

(Option 1) Autonomic dysreflexia (eg, throbbing headache, flushing, hypertension) is a life-threatening condition caused by sensory

stimulation that occurs in clients who have a spinal cord injury at T6 or higher. This is not the priority assessment as this client’s injury

is at L3. This client likely has acute urinary retention and needs catheterization.

 

(Option 2) Phenytoin toxicity commonly presents with neurologic manifestations such as gait disturbance, slurred speech, and

nystagmus. These are expected symptoms and therefore are not a priority.

 

(Option 3) A brain tumor can also cause increased intracranial pressure; clients report morning headache, nausea, and vomiting.

Dexamethasone (Decadron) can be prescribed short-term to decrease the surrounding edema. A tumor usually grows more slowly than a possible hematoma and is therefore not the priority assessment.

 

Educational objective:

Constant headache, decreased mental status, and sudden-onset emesis indicate increased intracranial pressure.

The nurse assesses a newly admitted adult client on a neurological inpatient unit. Which assessment findings require

immediate follow-up by the nurse? Select all that apply.

 

· 1. Cannot flex the chin toward the chest

· 2. Eyes move in opposite direction of head when head is turned to side

· 4. Pupils 8 mm in diameter bilaterally

 

The nurse performs a neurological assessment to evaluate for changes in motor and sensory functions by assessing pupils, cranial

nerves, and neuromuscular functioning. The neck should be supple and able to be flexed toward the chest. Nuchal rigidity requires

follow-up due to possible meningeal irritation related to infection (eg, meningitis) (Option 1).

 

A new-onset unilateral drift of a limb could indicate a stroke. The nurse assesses for other strokelike symptoms (eg, changing level of

consciousness, asymmetrical smile, garbled speech), activates the facility stroke protocol, and notifies the health care provider

accordingly (Option 3).

 

Normal pupils are 3-5 mm in diameter. Pupil dilation can be the result of medication use or neurological causes (eg, increased

intracranial pressure, brain herniation) (Option 4).

 

(Option 2) Oculocephalic reflex (doll’s eyes) is an expected finding indicating an intact brainstem. It is tested by rotating the head and watching for the eyes to move simultaneously in the opposite direction. The test is not performed if spinal trauma is suspected.

 

(Option 5) The normal finding in adults is an absent Babinski reflex (ie, toes point downward with stimulus to the sole). The presence

of Babinski reflex (ie, toes fan outward and upward with stimuli) is expected in infants up to age 1, but in an adult may indicate a

brain or spinal cord lesion.

 

Educational objective:

Abnormal neurological assessments include nuchal rigidity; new-onset unilateral drift of a limb; pupils <3 mm or >5 mm in diameter;

absent oculocephalic reflex; and presence of Babinski reflex in an adult client.

 

Assessment of a client with a history of stroke reveals that the client understands and follows commands but answers questions with

incorrect word choices. The nurse documents the presence of which communication deficit?

 

· 1. Aphasia (49%)

Aphasia refers to impaired communication due to a neurological condition (eg, stroke, traumatic brain injury). The term aphasia is

interchangeable with dysphasia, although aphasia is used more commonly. Receptive aphasia refers to impaired comprehension of

speech and writing. A client with receptive aphasia may speak full sentences, but the words do not make sense. The nurse should

speak clearly, ask simple “yes” or “no” questions, and use gestures and pictures to increase understanding.

 

Expressive aphasia refers to impaired speech and writing. A client with expressive aphasia may be able to speak short phrases but

will have difficulty with word choice (Option 1). The nurse should listen without interrupting and give the client time to form words.

A client may have one type of aphasia or a combination of both, and the severity will vary with the individual.

 

(Option 2) Apraxia refers to loss of the ability to perform a learned movement (eg, whistling, clapping, dressing) due to neurological

impairment.

 

(Option 3) Dysarthria is weakness of the muscles used for speech. Pronunciation and articulation are affected. Comprehension

and the meaning of words are intact, but speech is difficult to understand (eg, mumble, lisp).

 

(Option 4) Dysphagia refers to difficulty swallowing. The term dysphagia is often confused with dysphas/a. Clients with motor

deficits after a stroke may have dysphagia, which requires swallowing precautions to prevent aspiration.

 

Educational objective:

Aphasia refers to a neurological impairment of communication. Clients may have impaired speech and writing, impaired

comprehension of words, or a combination of both.

 

The nurse is providing discharge education for a postoperative client who had a partial laryngectomy for laryngeal cancer. The client

is concerned because the health care provider said there was damage to the ninth cranial nerve. Which statement made by the nurse

is most appropriate?

 

· 4. “This is the reason you are using a special swallowing technique when you eat and drink.” (58%)

 

Cranial nerve IX (glossopharyngeal) is involved in the gag reflex, ability to swallow, phonation, and taste. Postoperative partial

laryngectomy clients will need to undergo evaluation by a speech pathologist to evaluate their ability to swallow safely to prevent

aspiration. Clients are taught the supraglottic swallow, a technique that allows them to have voluntary control over closing the

vocal cords to protect themselves from aspiration. Clients are instructed to:

 

1. Inhale deeply

2. Hold breath tightly to close the vocal cords

3. Place food in mouth and swallow while continuing to hold breath

4. Cough to dispel remaining food from vocal cords

5. Swallow a second time before breathing

 

(Option 1) This would be considered “passing the buck.” The nurse should try to address the client’s concerns before calling the health care provider.

(Option 2) Cranial nerve VIII (vestibulocochlear) affects hearing and equilibrium, not swallowing.

 

(Option 3) The speech pathologist conducts a swallowing assessment early on to evaluate a client’s ability to swallow safely. This

consult is not done at discharge.

 

Educational objective:

Clients who undergo a partial laryngectomy are at increased risk for aspiration. As a result, they are taught a swallowing technique

(supraglottic swallow) to decrease this risk.

The nurse is caring for a client with an acute ischemic stroke who has a blood pressure of 178/95 mm Hg. The health care provider

prescribes as-needed antihypertensives to be given if the systolic pressure is >200 mm Hg. Which action by the nurse is most

appropriate?

 

· 2. Monitor the blood pressure (61%)

 

An ischemic stroke is a loss of brain tissue perfusion due to blockage in blood flow. Elevated blood pressure is common and

permitted after a stroke and may be a compensatory mechanism to maintain cerebral perfusion distal to the area of blockage. This

permissive hypertension usually autocorrects within 24-48 hours and does not require treatment unless the hypertension is extreme

(systolic blood pressure >220 mm Hg or diastolic blood pressure >120 mm Hg) or contraindicated due to the presence of another

illness requiring strict blood pressure control (eg, active ischemic coronary disease, heart failure, aortic dissection). A blood pressure

of 178/95 mm Hg should be monitored, along with the client’s other vital signs and status (Option 2).

 

(Option 1) The elevated blood pressure may be a protective measure to ensure tissue perfusion; therefore, the antihypertensive drug

should not be given unless indicated by the prescription parameters.

 

(Options 3 and 4) The as-needed prescription is appropriate in this case, so there is no reason to question it and call the health care

provider.

 

Educational objective:

Elevated blood pressure in the presence of an acute ischemic stroke is an expected finding and may be a protective measure to

maintain tissue perfusion.

 

A client with a ventriculoperitoneal shunt has a dazed appearance and grunting and has not responded to the caregiver for 10

minutes. Status epilepticus is suspected. Which nursing intervention should be performed first?

· 1. Administer rectal diazepam (30%)

This client is in status epilepticus, a serious and life-threatening emergency in which a client has been seizing for 5 minutes or

longer. Grunting and a dazed appearance are 2 common signs. A client with hydrocephalus (abnormal collection of cerebrospinal

fluid in the head) and a ventriculoperitoneal (VP) shunt is at a higher risk for seizures. Stopping seizure activity is the first nursing

priority. IV benzodiazepines (diazepam or lorazepam) are used acutely to control seizures. However, rectal diazepam is often

prescribed when the IV form is unavailable or problematic. Parents often get prescriptions for rectal diazepam and are advised to

administer a dose before bringing a child to the emergency department.

 

(Option 2) Stopping the status epilepticus is a priority over determining its cause through a neurologic assessment. Quickly obtaining

the oxygen saturation level and managing the airway are priority assessments.

 

(Option 3) Blood draw is needed for laboratory studies but is not a priority over stopping the seizure.

 

(Option 4) AVP shunt drains excess fluid in the brain down to the abdomen, where it is absorbed by the body. A CT scan can

accurately assess shunt malfunction. Any malfunction would need to be treated promptly to prevent future seizures and damage.

Finding the cause of the seizure is important and should be done as soon as seizing has stopped.

 

Educational objective:

Status epilepticus is a serious condition that could result in brain damage and death. Quickly stopping the seizure is the first nursing

priority as long as there is an adequate airway and the client is breathing. IV or rectal benzodiazepines (lorazepam or diazepam) are

used to rapidly control seizures.

 

The health care provider prescribes a multivitamin regimen that includes thiamine for a client with a history of chronic alcohol abuse.

The nurse is aware that thiamine is given to this client population for which purpose?

 

· 3. To prevent Wernicke encephalopathy (61%)

 

Clients with chronic alcohol abuse suffer from poor nutrition related to improper diet and altered nutrient absorption. Poor thiamine

intake and/or absorption can lead to Wernicke encephalopathy, a serious complication that manifests as altered mental status,

oculomotor dysfunction, and ataxia. Clients are prescribed thiamine to prevent this condition.

 

(Option 1) Thiamine is not used to reduce the blood alcohol level; there is no antidote for elevated blood alcohol levels. Supportive

 

measures are used until levels fall and the client is no longer experiencing acute withdrawal symptoms.

(Option 2) Gross tremors related to acute alcohol withdrawal are treated with benzodiazepines (eg, lorazepam, diazepam).

 

(Option 4) Seizures related to acute alcohol withdrawal are treated with benzodiazepines (eg, lorazepam, diazepam,

chlordiazepoxide). Thiamine is used to prevent encephalopathy. Encephalopathy may lead to seizures, but thiamine is not used to

treat seizures.

 

Educational objective:

A major complication of chronic alcohol abuse is encephalopathy related to poor thiamine absorption. It is critical that these clients

receive thiamine replacement. Wernicke encephalopathy can lead to more significant and progressive complications, including death.

The nurse assesses several clients using the Glasgow Coma Scale. Which scenario best demonstrates a correct application of this

scale?

· The nurse applies pressure to the nail bed, and the client tries to push the nurse’s hand away. The nurse scores motor response as “localization of pain.” (30%)

· The nurse asks the client what day it is and the client says “banana.” The nurse scores verbal response as “confused.” (21%)

· The nurse speaks with client and then the client’s eyes open. The nurse scores eye opening as “spontaneous.” (43%)

· The nurse walks in the room and the client states “Hi honey. How are you?” The nurse scores verbal response as “oriented.” (5%)

 

· Use best response for each category (range = 3-15).

· Coma: Does not open eyes, does not follow commands, and does not utter understandable words; Glasgow Coma Score (GCS) 3-8.

· Head injury classification: Mild, GCS 13-15; moderate, GCS 9-12; severe, GCS <8.

 

The Glasgow Coma Scale is used to determine level of consciousness. The nurse follows a standardized assessment to

determine the score of the client’s eye opening response, verbal response, and ability to obey commands through a motor

response. For the best motor response score, the nurse first verbally asks the client to obey a command. If there is no response, the

nurse next uses noxious stimuli (eg, nail bed pressure) and records the physical response. If the client tries to remove the painful

stimulus, it is recorded as “localizing” or moving toward the pain; whereas if the client retracts from the stimulus, it is recorded as

“withdrawal” (Option 1).

 

(Option 2) To ensure an accurate score in the verbal response category, the nurse must differentiate if the client is confused (eg,

answers “1955” when asked the year) or if a client uses inappropriate words.

 

(Option 3) To ensure an accurate eye opening score, the nurse must determine whether the client’s eyes open spontaneously (eg, no

prompting) or if a stimulus (eg, sound, pain) is needed.

 

(Option 4) Asocial, verbal client is not necessarily oriented. The nurse must assess orientation by specifically asking clients to state

their name, the time, and their location.

 

Educational objective:

The Glasgow Coma Scale is used to determine level of consciousness. The nurse follows a standardized assessment to determine

the score of the client’s eye opening response, verbal response, and ability to obey commands through a motor response.

 

The nurse is planning care for a client with suspected stroke who has just arrived at the emergency department with slurred speech,

facial drooping, and right arm weakness that began 1 hour ago. Which of the following interventions should the nurse anticipate

including in the initial plan of care? Select all that apply.

 

· 3. Obtain a STAT CT scan of the head

· 4. Perform a baseline neurologic assessment

· 5. Prepare to initiate alteplase within the next 3 hours

Strokes may be either ischemic or hemorrhagic. Ischemic stroke occurs when circulation to parts of the brain is interrupted by

occlusion of cerebral blood vessels by a thrombosis or embolus. Hemorrhagic stroke occurs when a cerebral blood vessel ruptures

and bleeds into the cranial vault. Both types of stroke result in brain tissue death without prompt treatment.

 

Aclient with stroke symptoms must have an immediate CT scan or MRI of the head to determine the type and location of the stroke

(Option 3). Determining exactly when symptoms began is essential for diagnosis and planning treatment.

 

Thrombolytic therapy (eg, alteplase, tissue plasminogen activator [tPA]) is used to dissolve blood clots and restore perfusion to brain

tissue in clients with an ischemic stroke unless contraindicated (eg, active bleeding, uncontrolled hypertension, aneurysm). It must

be administered within 4.5 hours from onset of symptoms (Option 5). A baseline neurologic assessment is essential for tracking

 

ongoing neurologic symptoms that indicate improvement or complications which guide later treatments (Option 4).

 

(Options 1 and 2) Consultation with a speech pathologist and providing the family with information about community resources are

important later but not during the initial (acute) phase of stroke management.

 

Educational objective:

The initial plan of care for a client with an acute stroke should include performing baseline neurologic assessment to begin monitoring

neurologic status trend, obtaining an immediate CT scan of the head to determine stroke type, and anticipating administration of

thrombolytics (if indicated) within 4.5 hours of symptom onset.

 

Aclient with a history of headaches is scheduled for a lumbar puncture to assess the cerebrospinal fluid pressure. The nurse is

preparing the client for the procedure. Which statement by the client indicates a need for further teaching by the nurse?

 

· 3. “I will need to lie on my stomach during the procedure.” (81%)

X 4. “The physician will insert a needle between the bones in my lower spine.” (1%)

 

Cerebrospinal fluid (CSF) is assessed for color, contents, and pressure. Normal CSF is clear and colorless, and contains a little

protein, a little glucose, minimal white blood cells, no red blood cells, and no microorganisms. Normal CSF pressure is 60-150 mm

H,0. Abnormal CSF pressure or contents can help diagnose the cause of headaches in complicated cases. CSF is collected via

lumbar puncture or ventriculostomy.

 

Prior to a lumbar puncture, clients are instructed as follows:

 

1. Empty the bladder before the procedure (Option 2)

 

2. The procedure can be performed in the lateral recumbent position or sitting upright. These positions help widen the space

between the vertebrae and allow easier insertion of the needle (Option 3).

3. Asterile needle will be inserted between the L3/4 or L4/5 interspace (Option 4)

4. Pain may be felt radiating down the leg, but it should be temporary (Option 1)

 

After the procedure, instruct the client as follows:

 

1. Lie flat with no pillow for at least 4 hours to reduce the chance of spinal fluid leak and resultant headache

2. Increase fluid intake for at least 24 hours to prevent dehydration

 

Educational objective:

Lumbar puncture can be performed with clients in the sitting position or positioned on the left side with the knees drawn up (fetal

position).

Anurse cares for a client with impairment of cranial nerve VII|. What instructions will the nurse provide the unlicensed assistive

personnel prior to delegating interventions related to the client’s activities of daily living?

 

· 4. “Make sure the items needed by the client are within reach.” (24%)

 

The client has an impairment of cranial nerve (CN) VIII, the vestibulocochlear (or auditory) nerve. Symptoms of impairment may

include loss of hearing, dizziness, vertigo, and motion sickness, which place the client at a high risk for falls. Therefore, when

instructing the unlicensed assistive personnel (UAP) about helping the client with activities of daily living, the nurse emphasizes the

need to keep items at the bedside within the client’s reach (Option 4).

 

(Option 1) Weakness of the shoulder muscle occurs with impairment of CN XI, the spinal accessory nerve. Impairment of CN VIII

does not affect shoulder strength.

 

(Option 2) Dysphagia may occur with impairment of CN IX (glossopharyngeal) and CN X (vagus), not CN VIII. Instructing the client to tuck the chin while eating is a technique for those who have difficulty swallowing.

 

(Option 3) Impairment of visual acuity occurs with disorders affecting CN II (optic). Because impairment of CN VIII does not affect

visual acuity, providing a detailed, step-by-step explanation of procedures may be helpful but is not the most appropriate instruction to

give the UAP.

 

Educational objective:

Impairment of cranial nerve (CN) VIII, the vestibulocochlear or auditory nerve, may cause dizziness, vertigo, loss of hearing, and

motion sickness. To assist the client with impairment of CN VIIl, needed items should be placed nearby to decrease the risk of the

client getting out of bed and falling.

 

Aclient with a T4 spinal cord injury has a severe throbbing headache and appears flushed and diaphoretic. Which priority

interventions should the nurse perform? Select all that apply.

 

X 1. Administer an analgesic as needed

· 2. Determine if there is bladder distention

· 3. Measure the client’s blood pressure

· 5. Remove constrictive clothing

Clients with a high (T6 or above) spinal cord injury are at risk for autonomic dysreflexia (autonomic hyperreflexia). It is an

uncompensated sympathetic nervous system stimulation. Classic signs include hypertension (up to 300 mm Hg systolic), throbbing

headache, diaphoresis above the level of injury, bradycardia (30-40/min), piloerection (“goose bumps”), flushing, and nausea. This is

a life-threatening condition that requires immediate intervention to prevent complications (eg, hypertensive stroke, seizures).

 

Clients with a spinal cord injury should have their blood pressure checked when they report a headache (Option 3).

 

The most common cause of autonomic dysreflexia is bladder irritation due to distention. The client needs to be catheterized or the

possibility of a kink in the existing catheter must be assessed (Option 2). Bowel impaction can also be a cause; a digital rectal

examination should be performed. Constrictive clothing should be removed to decrease skin stimulation (Option 5).

 

The primary health care provider should be notified. An alpha-adrenergic blocker or an arteriolar vasodilator (eg, nifedipine) may be

prescribed.

 

(Option 1) Headaches associated with autonomic dysreflexia are typically due to severe hypertension and often resolve after blood pressure has been treated.

 

(Option 4) The client should have the head of the bed elevated 45 degrees or high Fowler’s to lower blood pressure. The Sims’

position is flat and side-lying.

 

Educational objective:

Autonomic dysreflexia is a life-threatening condition in a client with high spinal cord injury. Classic signs/symptoms include severe

hypertension, throbbing headache, diaphoresis, bradycardia, flushing, and piloerection. Emergency treatment includes correcting the

cause (check bowel or bladder distention), removing tight clothing, and raising the head of the bed.

 

A client is brought to the emergency department by emergency medical services with a flaccid right arm and leg and lack of verbal

response. The stroke alert team is initiated. The nurse takes which priority action?

 

· 3. Maintain patent airway (83%)

 

A flaccid extremity and change in verbal ability are symptoms of a stroke, which is considered an emergency. Clients with stroke

symptoms are immediately triaged using a special team and set of tools to determine the correct course of action with the goal of

preventing further brain damage. In any emergency, the first priority nursing action is to maintain a patent airway (Option 3).

Depending on the mechanism of injury, the symptoms may include changes in airway clearance, which is a priority.

 

The nurse, or another member of the emergency department or stroke alert team, will prepare the client for an immediate head CT

scan to rule out a hemorrhagic stroke and determine the location and extent of the injury (Option 4). This person will also ensure that

the client has 2 large-bore IV lines for rapid infusion of fluids or medications as needed (Option 2).

 

(Option 1) It is vital to determine the onset of symptoms as thrombolytic medications are used in a short time frame (typically within

4.5 hours of onset). Thrombolytic medications are used only in ischemic strokes, so the head CT must be completed to confirm the

type of stroke (ischemic versus hemorrhagic). With all of these interventions, the priority nursing actions remain the same: ABC –

airway, breathing, and circulation.

 

Educational objective:

In any emergency, the primary nursing interventions are the ABCs. A patent airway should be maintained while other care is provided

and throughout the emergency treatment process.

The nurse is caring for a client in the immediate postoperative period following a carotid endarterectomy. The client is drowsy with

slurred speech. Which assessment finding would cause the nurse to notify the healthcare provider immediately?

 

· wt 3. Left arm drift during bilateral arm extension (53%)

 

Acarotid endarterectomy is a surgical procedure performed to remove plaque from the carotid artery to improve cerebral

perfusion, The nurse must closely assess for signs of new or worsening alterations in neurologic status, as surgical manipulation of arteries and blood flow increases the risk of stroke.

 

Monitoring the client’s neurologic status postoperatively can be challenging, as the effects of anesthesia degrade the neurologic

examination. Nurses should use the FAST acronym to assess for stroke:

 

· Facial drooping: Numbness or droopiness on one side of the face

· Arm weakness: Weakness or drifting of one arm when raised to shoulder level (Option 3)

· Speech difficulties: Slurring of words, incomprehensible speech, inability to understand others

· « Time: Notation of the time of symptom onset, which is critical for guiding treatment

 

(Option 1) Diminished gag reflex is common after anesthesia and endotracheal tube removal. The gag reflex should return as the

client awakens.

 

(Option 2) Individuals recovering from anesthesia may have alterations in mood or affect (eg, agitation, anxiety, tearfulness) that will

resolve as anesthesia wears off.

 

(Option 4) Drowsiness and somnolence during purposeful interactions (ie, following commands) are expected after anesthesia.

 

Educational objective:

Following a carotid endarterectomy, the client should be monitored for alterations in mental status that are unexpected in the context

of typical postanesthesia symptoms (eg, diminished gag reflex, altered affect, drowsiness). The FAST assessment (Facial drooping,

Arm weakness or drift, Speech difficulties, Time) assists with identifying alterations that may indicate stroke.

 

The nurse is assessing the cranial nerves and begins testing the facial nerve (cranial nerve VII). Which direction should the nurse

give the client to test this cranial nerve?

 

· 4. “Raise your eyebrows, smile, and frown.” (84%)

The facial nerve, cranial nerve VII, is tested by assessing exaggerated facial movements. The client is directed to raise the eyebrows,

furrow the eyebrows, draw up the cheeks in a large smile, pull the cheeks down in a frown, and open the lips to show the teeth. Any

asymmetrical movements are documented, and if unexpected, the health care provider is notified.

 

(Option 1) Cranial nerve | is the olfactory sensory nerve. This nerve is tested by having the client identify a readily recognized odor.

 

(Option 2) Cranial nerve III is a motor nerve of the eye, which is tested by having the client track an object, such as a finger, through

the fields of vision.

 

(Option 3) Cranial nerve II is the optic nerve and is a sensory nerve. It is assessed by testing the fields of vision for the client’s ability

to see objects in the field.

 

Educational objective:

The facial nerve, cranial nerve Vil, is tested by assessing exaggerated facial movements. The client is directed to raise the eyebrows,

furrow the eyebrows, draw up the cheeks in a large smile, pull the cheeks down in a frown, and open the lips to show the teeth.

 

The emergency department nurse assesses a client involved in a motor vehicle accident who sustained a coup-contrecoup head

injury. Which assessment finding is consistent with injury to the occipital lobe?

 

· 2. Deficits in visual perception (81%)

 

A coup-contrecoup head injury occurs when the head strikes an object and the brain receives an injury under the area of impact

(coup), after which it rebounds to the opposite side of the skull and sustains injury on that side as well (contrecoup). This type of injury is common in motor vehicle accidents and shaken baby syndrome. Visual processing occurs in the occipital lobe.

 

(Option 1) The rate and depth of respirations are regulated by the medulla oblongata at the base of the brainstem.

 

(Option 3) Expressive aphasia, the inability to express spoken words, occurs after a transient ischemic attack or stroke. This will

occur if the frontal lobe (Broca aphasia) or temporal lobe (Wernicke aphasia) is injured.

 

(Option 4) Inability to recognize being touched is indicative of injury to the parietal lobe of the brain.

 

Educational objective:

Coup-contrecoup head injuries are common in motor vehicle accidents and shaken baby syndrome. Damage to the occipital lobe of

 

the brain during coup-contrecoup head injury will result in visual disturbances.

Acclient is being admitted for a potential cerebellar pathology. Which tasks should the nurse ask the client to perform to assess if

cerebellar function is within the defined limits? Select all that apply.

· 4. Touch each finger of one hand to the hand’s thumb

· 5. Walk heel-to-toe

The cerebellum is involved in 2 major functions: coordination of voluntary movements and maintenance of balance and posture.

Maintenance of balance is assessed with gait testing and includes watching the client’s normal gait first and then the gait on heel-to-

toe (tandem), on toes, and on heels (Option 5).

 

Coordination testing involves the following:

 

1. Finger tapping — ability to touch each finger of one hand to the hand’s thumb (Option 4).

2. Rapid alternating movements — rapid supination and pronation

3. Finger-to-nose testing — clients touch the clinician’s finger and then their own nose as the clinician’s finger varies in location

4. Heel-to-shin testing — client runs each heel down each shin while in a supine position

 

(Option 1) This is a test of sensory function, specifically fine touch (graphesthesia). Other tests for this include identifying an object

in the hand (stereognosis) and two-point discrimination.

 

(Option 2) Shrugging the shoulders against resistance (as well as turning the head against resistance) is a test for cranial nerve (CN)

XI (spinal accessory).

 

(Option 3) In a client who has an intact gag reflex, the ability to swallow water helps to assess CN IX (glossopharyngeal) and CN X

(vagus). The nurse can also observe for a symmetrical rise of the soft palate and uvula by asking the client to say “ah.”

 

Educational objective:

The cerebellum is involved in coordination of voluntary movements and maintenance of balance and posture. Balance is assessed

with heel-to-toe gait testing. Coordination is assessed with finger tapping, rapid alternating movements, finger-to-nose testing, and

heel-to-shin testing.

 

An adult client with altered mental status and fever has suspected bacterial meningitis with sepsis. Blood pressure is 80/60 mm Hg.

Which prescribed intervention should the nurse implement first?

 

· Infuse bolus of IV normal saline (67%)

 

Meningitis is an inflammation of the meninges covering the brain and spinal cord. The key clinical manifestations of bacterial

meningitis include fever, severe headache, nausea/vomiting, and nuchal rigidity. Other symptoms include photophobia, altered

mental status, and other signs of increased intracranial pressure (ICP).

 

In a hypotensive client with sepsis, the priority of care is fluid resuscitation to increase the client’s blood pressure (Option 2).

In addition to IV fluid administration, interventions and prescriptions for a client with sepsis and meningitis may include:

 

· Administer vasopressors.

· Obtain relevant labs and blood cultures prior to administering antibiotics.

· Administer empiric antibiotics, preferably within 30 minutes of admission (Option 1). This client will continue to decline without antibiotic therapy.

· Prior to a lumbar puncture (LP), obtain a head CT scan as increased ICP or mass lesions may contraindicate a LP due to the

risk of brain herniation (Option 4).

· Assist with a LP for cerebrospinal fluid (CSF) examination and cultures (Option 3). CSF is usually purulent and turbid in clients with bacterial meningitis. CSF cultures will allow for targeted antibiotic therapy.

 

Educational objective:

 

For bacterial meningitis with sepsis, fluid resuscitation is the priority. Blood cultures should be drawn before starting antibiotics. After

a head CT scan is performed to rule out increased intracranial pressure and mass lesions, cerebrospinal fluid cultures should be

drawn via lumbar puncture.

The nurse is caring for a client diagnosed with Broca aphasia due to a stroke. Which of the following deficits would the nurse correctly

attribute to Broca aphasia? Select all that apply.

 

· 2. Client is easily frustrated while attempting to speak

· 5. Client’s speech is limited to short phrases that require effort

 

Broca (expressive) aphasia is a nonfluent aphasia resulting from damage to the frontal lobe. Clients with Broca aphasia can

comprehend speech but demonstrate speech difficulties. The speech pattern often consists of short, limited phrases that make sense

but display great effort and frequent omission of smaller words (eg, “and,” “is,” “the”) (Option 5).

 

Clients with Broca aphasia are aware of their deficits and can become frustrated easily (Option 2). In comparison, clients with

Wernicke (receptive) aphasia are unaware of their speech impairment.

 

(Option 1) Trouble swallowing, often identified by coughing and gasping when eating and drinking, is dysphagia, which is not related

to Broca aphasia.

 

(Option 3) Clients with damage to multiple language areas of the brain may develop global aphasia, resulting in the inability to read,

write, or understand speech. This is the most severe form of aphasia.

 

(Option 4) Clients with damage to the temporal portion of the brain may develop Wernicke (fluent) aphasia (ie, the inability to

comprehend the spoken and/or written word) and exhibit a long, but meaningless, speech pattern.

 

Educational objective:

Damage to the frontal lobe of the brain may cause Broca (expressive) aphasia. Clients with this condition demonstrate effortful and

sensible speech characterized by short, limited sentences, with retained ability to comprehend speech. This impairment often causes

clients with Broca aphasia to be frustrated when speaking.

 

The nurse is caring for a client diagnosed with Guillain-Barré syndrome (GBS) after a recent gastrointestinal (Gl) illness. Monitoring

for which of the following is a nursing care priority for this client?

 

· 3. Inability to cough or lift the head (65%)

GBS Is an acute, immune-mediated polyneuropathy that is most otten accompanied by ascending muscle weakness and absent

deep-tendon reflexes. Many clients have a history of antecedent respiratory tract or Gl infection. Lower-extremity weakness

progresses over hours to days to involve the thorax, arms, and cranial nerves. However, neuromuscular respiratory failure is the

most life-threatening complication.

 

Early signs indicating impending respiratory failure include:

 

· Inability to cough

· Shallow respirations

· Dyspnea and hypoxia

· Inability to lift the head or eye brows

 

Assessing the client’s pulmonary function by serial spirometry is also recommended. Measurement of forced vital capacity (FVC)

is the gold standard for assessing ventilation; a decline in FVC indicates impending respiratory arrest requiring endotracheal

intubation.

 

(Option 1) Severe autonomic dysfunction can present as diaphoresis and facial flushing.

 

(Option 2) The client with GBS is also at risk for paralytic ileus, which is related to either immobility or nerve damage. As a result, the nurse should monitor for the presence hypoactive/absent bowel sounds.

 

(Option 4) Clients with GBS are at risk of developing deep venous thrombosis due to lack of ambulation and should receive

pharmacologic prophylaxis (heparin) and support stockings.

 

Although symptoms in options 1, 2, and 4 represent a progressive illness and are important to communicate to the health care

provider promptly, they are not the highest priority compared to impending respiratory failure.

 

Educational objective:

Respiratory distress is a potential complication of progressing paralysis in clients with Guillain-Barré syndrome. The nurse should

prioritize and monitor for the presence of this complication. Measurement of serial spirometry (FVC) is the gold standard for

silt

 

A highly intoxicated client was brought to the emergency department after found lying on the sidewalk. On admission, the client is

awake with a pulse of 70/min and blood pressure of 160/80 mm Hg. An hour later, the client is lethargic, pulse is 48/min, and blood

pressure is 200/80 mm Hg. Which action does the nurse anticipate taking next?

 

 

· 3. Have CT scan performed to rule out an intracranial bleed (41%)

Cushing’s triad is related to increased intracranial pressure (ICP). Early signs include change in level of consciousness. Later signs

include bradycardiaincreased systolic blood pressure with a widening pulse pressure (difference between systolic and

diastolic), and slowed irregular (Cheyne-Stokes) respirations. Cushing’s triad is a later sign that does not appear until the ICP is

increased for some time. It indicates brain stem compression.

 

The skull cannot expand after the fontanels close at age 18 months, so anything taking up more space inside the skull (eg, hematoma,

tumor, swelling, etc.) is a concern for causing pressure on the brain tissue/brain stem and potential herniation. In this scenario, hidden

 

head trauma causing an intracranial bleed must be ruled out with diagnostic testing. The client’s intoxication could blunt an accurate

history or presentation of a head injury.

 

(Option 1) Atropine is used to stimulate the sinoatrial node in bradycardia with systemic symptoms. An electrocardiogram (ECG)

should be obtained prior to administering atropine. In this client, there is no evidence of a cardiac etiology or systemic symptoms of

poor perfusion/circulation from the bradycardia.

 

(Option 2) Nifedipine (Procardia) is a calcium channel blocker that is a potent vasodilator. However, all components must be

considered in this scenario as to the etiology of the hypertension rather than just treating that sign. Ruling out a cerebral cause of the

hypertension is most important.

 

(Option 4) The nurse would continue to perform neurologic assessments (including GCS). However, it is more important to obtain

appropriate diagnostic tests and initiate treatment for the changing neurologic symptoms than to just monitor and document. In

addition, the nurse should be performing these assessments more frequently than hourly in this rapidly changing client.

 

Educational objective:

Cushing’s triad/reflex indicates increased intercerebral pressure. Classic signs include bradycardia, rising systolic blood pressure,

widening pulse pressure, and irregular respirations (such as Cheyne-Stokes).

 

The nurse is caring for a client following a transsphenoidal hypophysectomy. Which clinical findings would the nurse recognize as

signs that the client may be developing diabetes insipidus? Select all that apply.

 

· 2. Excess oral water intake

· 3. High urine output

· 4. Increased serum osmolality

 

Transsphenoidal hypophysectomy is the surgical removal of the pituitary gland, an endocrine gland that produces, stores, and

excretes hormones (eg, antidiuretic hormone [ADH], growth hormone, adrenocorticotropic hormone). Clients undergoing

hypophysectomies are at risk for developing neurogenic diabetes insipidus (Dl), a metabolic disorder of low ADH levels. ADH

promotes water reabsorption in the kidneys. Therefore, loss of circulating ADH results in massive diuresis of dilute urine. Clinical

manifestations associated with DI include:

 

· Decreased urine specific gravity (<1.003) (Option 5)

· Elevated serum osmolality (>295 mOsm/kg [295 mmol/kg]) (Option 4)

· Hypernatremia (>145 mEq/L [145 mmol/L]) (Option 1)

· Hypovolemia and potential hypotension

· Polydipsia (Option 2)

· Polyuria (2-20 L/day) (Option 3)

 

Educational objective:

Diabetes insipidus (Dl) is a metabolic disorder of decreased antidiuretic hormone, which is responsible for water retention in the

kidneys. Dl is often related to a preceding trauma, pituitary tumors, or neurosurgery (eg, hypophysectomy). Clinical manifestations of

DI include polyuria, polydipsia, hypernatremia, hypovolemia, increased serum osmolality, and decreased urine specific gravity.

Aclient with a C3 spinal cord injury has a headache and nausea. The client’s blood pressure is 170/100 mm Hg. How should the

nurse respond initially?

 

· 4, Palpate the client’s bladder (62%)

 

Autonomic dysreflexia is an acute, life-threatening response to noxious stimuli, which clients with spinal cord injuries above T6

are unable to feel. Signs and symptoms include hypertension, bradycardia, a pounding headache, diaphoresis, and nausea. It

is essential that the nurse assess for and remove noxious stimuli to prevent a stroke (Option 4).

 

Noxious stimuli may include:

· Bladder distention (eg, obstructed urinary catheter, neurogenic bladder)

· Fecal impaction

· Tight clothing (eg, shoelaces, waistbands)

 

(Options 1 and 2) Hypertension, headache, and nausea due to uncontrolled sympathetic activity will resolve once the cause is

identified and removed.

 

(Option 3) Lowering the head of the bed would increase blood pressure. The head of the bed should be raised to lower the blood

pressure.

 

Educational objective:

Autonomic dysreflexia is an acute, life-threatening response to noxious stimuli (eg, bladder distention, tight clothing) in clients with

spinal cord injuries above T6. Signs and symptoms include hypertension, bradycardia, a pounding headache, diaphoresis, and

nausea. The nurse must immediately identify and remove noxious stimuli to prevent a stroke and resolve symptoms.

 

Glasgow Coma Scale

(E)ye opening

(Maximum = 4)

 

4 – Spontaneous (open with blinking at baseline)
  3 – To speech
  2 – To pain only
  1-None
  (C – Not assessable [eg, trauma, edema])
 
(V)erbal response

(Maximum = 5)

 

5 – Oriented
  4 – Confused (converses but confused, disoriented)
  3 – Inappropriate (inappropriate words)
  2 – Incomprehensible (sounds, no words)
  1 – None
  (T – Not assessable [intubated])
   

 

 

 

 

 

 

 

(M)otor response

(Maximum = 6)

 

 

6 – Obeys commands for movement

  5 – Localizes to pain
  4 – Withdraws from pain
  3 – Flexion in response to pain (decorticate posturing)
  2 – Extension in response to pain (decerebrate posturing)
  1- None

 

 

Glasgow Coma Scale
Eye opening

 

Spontaneously 4

To speech 3

To pain 2

None 1

Verbal response

 

Oriented 5

Confused 4

Inappropriate 3

Incomprehensible 2

None 1

 

 

 

 

 

 

 

 

 

 

 

Motor response

 

Obeys commands 5

Localizes to pain 4

Withdraws from pain 4

Flexion to pain 3

Extension to pain 2

None 1

  Maximum score 15

 

 

 

 

 

 

 

 

 

©uUWorld

 

 

 

Emergency/trauma assessment

 

Primary survey

 

Secondary survey

 

A – Airway: Clear airway,

Stabilize cervical spine

F — Full set of vital signs/Focused

interventions: monitor ECG, place

tubes & drains

B — Breathing: Give oxygen,

prepare to intubate

 

G — Give comfort: Assess & treat pain
Cc -— Circulation: Check pulse &

heart rate, control bleeding

 

H — Head-to-toe assessment/History:

Use SAMPLE -— Signs/Symptoms,

Allergies, Medications, Past health

history, Last meal, Events preceding

 

D — Disability: Check level of

consciousness, assess using

Glasgow Coma Scale

|- Inspect posterior: Logroll & palpate

 

 

E — Exposure: Remove clothing,

prevent heat loss

 

 

 

 

 

 

 

 

 

 

 

 

 

 

 

 

 

 

 

 

 

 

 

 

 

 

 

 

 

 

 

 

U world — Neurological pharmacology .

A client with Parkinson disease is prescribed carbidopa-levodopa. Which of the following instructions should the nurse include with

the client’s discharge teaching? Select all that apply.

· 1. “Change positions slowly and sit on the side of the bed before standing.”

 

· 2” This medication takes several weeks to reach› maximum benefit.”

 

· 3″Your urine and saliva may turn reddish-brown, but this is. not harmful.”

 

Parkinson disease (PD) is characterized by decreased dopamine levels, uncontrolled acetylcholine, and formation of abnormal

protein clusters (Lewy bodies) in the brain. PD causes both physical and neurological (eg, mood alterations, dementia) symptoms.

 

Carbidopa-levodopa is a combination antiparkinsonian medication used to reduce physical symptoms of PD by increasing

dopamine levels in the brain. Levodopa is converted to dopamine in the brain but is largely metabolized before reaching the brain.

Carbidopa does not have a therapeutic effect on PD but prevents breakdown of levodopa before reaching the brain, which makes levodopa more effective.

Client teaching for carbidopa-levodopa includes:

 

· implementing fall precautions (e.g., changing positions slowly, removing rugs), as orthostatic hypotension is. a common side effect (Option 1)

· Knowing that carbidopa-levodopa takes several weeks to reach its maximum effectiveness (Option 2)

· Understanding that harmless discoloration (e.g, red brown black) of secretions (e.g. urine, perspiration, saliva) may occur while taking carbidopa-levodopa (Option 5)

· Avoiding high-protein meals, which interfere with the absorption of carbidopa-levodopa

 

(Option 3) Dyskinesia (eg, facial or eyelid twitching, tongue protrusion, facial grimacing) may indicate overdose’ or toxicity of carbidopa-levod6pa and’ should be reported immediately to the health care provider.

(Option 4) Carbidopa-levodopa. often‹ decreases, but does not eliminate, tremor and rigidity.

 

Educational objective:

Carbidopa-levodopa is a medication used to reduce symptoms of tremor and reduce in clients with Parkinson disease: Teach clients that the medication takes several weeks to become effective; urine„ perspiration, or saliva. discoloration is a common. side effect; and fall precautions should be implemented ‘ for client safety.

Levodopa. is administered in combination with carbidopa to slow down the progression of Parkinson’s’ disease. This mediation functions by increasing the amount› of dopamine in the brain. Levodopa/Carbidopa is associated with the development of certain side effects, including dyskinesia, nausea and vomiting, cardiac dysrhythmias, psychosis, and postural hypotension. Some patients taking levodopa/carbidopa have reported feeling uncontrolled movements, an off and on phenomenon, and may experience darkened sweat or urine.

 

A client is receiving scheduled doses of carbidopa-levodopa. The nurse evaluates the medication as having the intended effect if which finding is noted?

· Improvement in spontaneous activity (65°/o)

 

 

 

 

Parkinson disease is caused by low levels of dopamine in the brain., Levodopa is converted to dopamine in the brain but much of this drug is metabolized before reaching the bran. Carbid6pa helps prevent the breakdown of levodopa before it can reach the brain and take effect. This combination medication is particularly effective in treating bradykinesia (generalized slowing of movement).

Tremor and rigidity may also improve to some extent. Carbidopa levodopa (Sinemet) once started should never be stopped suddenly as this can lead to akinetic crisis (complete loss of movement). However, prolonged use can also result in dyskinesias (spontaneous involuntary movements) i and on/off periods when the medication will start or stop working unpredictably.

(Option 1) Carbidopa-levodopa does not improve memory. Medications for the treatment of Alzheimer disease, such as donepezil and rivastigmine, are used to improve cognition and memory

(Options 3 and 4) Orthostatic hypotension and neuropsychiatric disturbances (e.g. confusion. hallucinations, delusions, agitation psychosis) are serious and important adverse effects of carbidopa-levodopa. Health care providers usually start the medications at low doses and gradually increase them to prevent these effects.

Educational objective:

The combination medication carbidopa-levodopa is most helpful for treating bradykinesia in Parkinson disease and can also improve tremor and rigidity to some extent. It is started in low doses to prevent orthostatic hypotension and neuropsychiatric adverse effects. Carbidopa-levodopa once started should never be stopped sudden as doing so can lead to akinetic crisis (complete loss of movement).

The clinic nurse is instructing a client who is newly prescribed transdermal scopolamine to prevent motion sickness during an upcoming vacation. on a cruise she. Which of the following statements made by the nurse are appropriate? Select. all that. apply.

· Dispose of the patch out of children and pets

· Make sure to remove the old patch before applying a new one

· Place the patch on a hairless, clean, dry area behind the ear

· Wash your hand with soap and water handling the patch

 

 

Scopolamine is an anticholinergic medication used to prevent nausea and vomiting from motion sickness and as an adjunct to anesthesia to contra secretions. Transdermal scopolamine is placed on a hairiest. clean, deg area behind the ear for proper absorption (Option. 4) Clients should be instructed to:

· Apply the patch 4 hours before starting travel to allow for absorption and medication onset (Option 1). Transdermal patches have a slower onset but a longer duration of action.

· Replace the patch every 72 hours as prescribed to ensure continuous medication delivery.

 

· Remove and discard the old patch before placing a new one to prevent accidental overdose (Option 3).

 

· Dispose of the old patch out of reach of children and pets to avoid accidental ingestion (Option 2)

 

· Wash hands with soap and water after handling the patch› to avoid inadvertent drug absorption or contact with the eyes (Option 5)

Educational objective

To prevent motion sickness, transdermal scopolamine should be applied to a hairless, clean, dry area behind the ear -4 hours prior to travel. Clients should change the patch every 72 hours, discard old patches out of reach of children and pets, and wash hands after handling patches. The old patch must be removed before a new one is placed

 

 

 

The nurse moves a finger in a horizontal and vertical motion in› front of the client’s face while directing the client to follow the finger with the eyes. Which cranial nerves is the nurse assessing? Select ally that apply.

 

· III

· Iv

· vi

 

The oculamotor (cranial nerve lii), trochlear (cranial: nerve IV), and abducens (cranial nerve VI) are motor nerves of the eye that are tested by having the client track an object,. such! as a finger, through the fields of vision. The oculomotor nerve is also tested by checking for pupillary constriction. and accommodation (constriction with near vision).

Deficits in cranial . nerves lit. iV, and’ VI. can. include deconjugate gaze (eyes do not move together), nystagmus (fine, rapid jerking eye movements.), or ptosis (drooping of the eyelid).

(Option 1) Cranial nerve li is the optic nerve and a sensory nerve. This nerve is assessed by testing the fields of vision for the client’s ability to see objects in the fields In contrast to cranial nerves lii, IV, and’ VI, the client does not track the object in the fields of vision., but instead keeps the eyes fixed and uses the peripheral. vision to recognize objects or deficits in the field of vision.

(Option 4) Cranial nerve V is the trigeminal nerve. The sensory portion of this nerve is assessed by testing sensation at the ophthalmic (forehead), maxillary (cheekbone), and mandibular Law line) branches by light touch Corneal sensation is also a portion of the trigeminal nerve, but this is typicity not tested by the nurse.

Educational. objectives

Motor function of the eyes is tested by having the client track an object through the fields of vision and documenting any abnormalities in› eye movement that represent cranial nerves lii, IV, and VI. The health care provider is then notified of the findings.

 

 

The nurse ä caring for a client after a motor vehicle accident. The client’s injuries include 2 fractured ribs and a concussion. The nurse notes which of the following as expected neurological changes for the client with a concussion? Select all that apply. .

· 2: Brief loss of consciousness

· 3. Headache

 

· 5. Retrograde amnesia

 

 

 

A concussion. is considered. a minor traumatic brain injury and results from blunt force or an acceleration/deceleration head injury. Typical signs of concussion include:

1. A brief disruption in level of consciousness

2, Amnesia regarding the event (retrograde amnesia)

3. Headache

 

These clients should be observed closely by family members and not participate in› strenuous or athletic activities for 1—2. days. Rest and a light diet are encouraged during this time.

(Options II and 4)‘ The following manifestations indicate more serious brain injury and are not expected with simple concussion:

 

· Worsening headaches and vomiting (indicate high intracranial pressure)

· Sleeping and/or confusion (indicate. high intracranial pressure)

· Visual changes

· Weakness or numbness of part of the body

 

Educational objective:

Expected neurological changes with a concussion include brief loss of consciousness, retrograde amnesia, and headache. These

clients should be observed I closely by family members and not participate in strenuous or athletic activities for 1-2. days

The’ nurse ‘ is caring for a client with bell palsy. Which of the following assessment findings does the nurse expect† Select all that apply?

· Change in lacrimation on the affected side.

· Flattening of the nasolabial fold

· Inability to simile symmetrically

 

Bell palsy is peripheral unilateral facial paralysis characterized I by in11ammation of the facial nerve (cranial ‘ nerve VII) h the absence of a stroke or other causative agent/disease. Paralysis of the motor fibers innervating the facial muscles results in flaccidity on the affected side.

Manifestations of Bell palsy include:

 

· Inability to completely close the eye on the affected side

· Alteration in› tear production (e.g. decreased tearing with extreme dryness, excessive tearing) due to weakness of the lower eyelid muscle (Option 1)

· Flattening of the nasolabial fold on the side of the paralysis (Option 3)

· Inability to smile or frown symmetrical} (Option 4)

 

Alteration in the sensory fibers can cause loss of taste on the anterior two-thirds of the tongue.

 

(Options 2 and 5) Electric shock—like pain in the lips and gums and severe pain along the cheekbone are symptoms of trigeminal. neuralgia (cranial. nerve V). With. Bell palsy, the trigeminal nerve may become hypersensitive. and cause facial pain but this is uncommon and typically more indicative of trigeminal neuralgia.

Educational objective:

Bell palsy is unilateral facial paralysis due to inflammation of the facial nerve that is characterized by inability to close the affected eye completely, changes in tear production, facial droop, and asymmetrical smil4 or frown.

 

 

 

The nurse is caring for a client following a transsphenoidal hypophysectomy. Which clinical findings would the nurse recognize as signs that the client may be developing diabetes insipidus? Select all that apply.

 

· 2. Excess oral water intake

 

· 3. High urine output:

 

· Increased serum osmolality

 

 

Transsphenoidal hypophysectomy is the surgical removal of the pituitary gland, an endocrine gland that produces, stores, and excretes hormones (eg, antidiuretic hormone [ADH], growth hormone, adrenocorticotropic hormone). Clients undergoing hypophysectomies are at risk for developing neurogenic diabetes insipidus. (DI),’ a metabolic disorder of low ADH levels. ADH promotes water reabsorption in the kidneys. Therefore, loss of circul8ting ADHI results in massive diuresis of dilute urine. Clinical manifestations associated with DI include:

· Decreased urine specific gravity (<1.003) (Option 5)

· Elevated serum osmolality (>295 mOsm/kg [295 mmol/kg]) (Option 4)

· Hypernatremia (>145 mEq/L (145 mmol/L]) (Option I)

· Hypovolemia and potential hypotension ›

· Polydipsia (Option 2)

· Polyuria (2-20 L/day) (Option 3)

 

Educational ‘ objective:

Diabetes insipidus. (DI) is a metabolic disorder of decreased. antidiuretic hormone, which. is responsible. for water retention in the kidneys. DII is often: related to a preceding trauma, pituitary tumors, or neurosurgery (eg, hypophysectomy). Clinical manifestations of DI include polyuria, polydipsia, hypernatremia, , hypovolemia,. increased! serum osmolality, and decreased urine specific gravity.

 

 

A client with blunt head injury if admitted I for observation, Including hourly neurology checks. At. 01 :00 AM, the client reports a headache; the nurse’ obtains a normal neurologic assessment and administers the PRN acetaminophen. i At 02’:00 ‘ AM, the client appears to be sleeping. Which. action should the nurse take?

· Arouse the client and ask what the current month is (55°/o)

 

 

 

Serial neurologic assessments are important as neurologic abnormalities are often initially subtle, making it important to note the trend. Interventions for neurologic issues are most effective when made early. A neurologic assessment includes:

1. Glasgow Coma Scale (GCS)—best. eye, verbal, and motor responses. Best verbal I response assesses orientation to person, place, and. time (time is the most sensitive).

2. Pupils-equal, I round, response to light, and. accommodates (PERRLA)

Motor—strength, and movement ink all four extremities

4, Vital signs—especially any signs of Cushing’s triad of bradycardia., bradypnea/abnormal. breathing pattern and widening pulse pressure (the difference between systolic and diastolic blood pressure readings). The nurse is assessing for signs of increased intracranial pressure (ICP)’.

This client. is not admitted in the hospital to get a good night sleep. The client. is admitted due to the need for serial neurologic assessments by a professional:’ nurse, and that is the priority.

(Option 2) Although pain relief has probably been achieved, this option› does not reflect. any neurologic assessment. One of the early signs of increased ICP is change in level of consciousness. This option does not assess the client’s arousability.

(Option 3) Checking the respiratory rate and characteristics is part of the neurologic assessment. However, this alone is insufficient for assessing a neurologic status after a blunt head injury.

(Option 4)‘ It is good to awaken the client, but paresthesia, is part of a neurovascular assessment. Neurovascular assessment, commonly known as the 5 Ps, consists of paresthesia. pain, pallor, paralysis… and pulselessness A neurovascular assessment is used when circulation is a primary concern. It is not the primary neurologic assessment needed in a closed head injury.

Educational objective.

Clients must be awakened for a prescribed, necessary neurologic assessment. A neurologic assessment consists of GCS, PERRLA, movement and strength of the extremities, and vital signs.

 

 

The nurse receives report for 4 clients. in the emergency department. Which client. should be seen first?

 

· 70-year-old with atrial fibrillation and a closed-head injury waiting for brain imaging who reports a headache’ and had emesis is of 200 ml (41 °/o)

A client with a neurological injury (eg, head trauma, stroke) is at risk for cerebral edema and increased intracranial pressure (ICP), a life-threatening situation. The client with atrial fibrillation may also be taking anticoagulants (eg, warfarin, rivaroxaban, apixaban, dabigatran), making a life-threatening intracranial bleed even more dangerous. The nurse should perform a neurologic assessment (eg, level of consciousness, pupil response, vital signs) immediately.

 

(Option 1) Autonomic dysreflexia (eg, throbbing headache, flushing, hypertension) is a life-threatening condition caused by sensory stimulation that occurs in clients who have a spinal cord injury at T6 or higher. This is not the priority assessment as this client’s injury is at L3. This client likely has acute urinary retention and needs catheterization.

 

(Option 2) Phenytoin toxicity commonly presents with neurologic manifestations such as gait disturbance, slurred speech, and nystagmus. These are expected symptoms and therefore are not a priority.

 

(Option 3) A brain tumor can also cause increased intracranial pressure; clients report morning headache, nausea, and vomiting. Dexamethasone (Decadron) can be prescribed short-term to decrease the surrounding edema. A tumor usually grows more slowly than a possible hematoma and is therefore not the priority assessment.

 

Educational objective:

Constant headache, decreased mental status, and sudden-onset emesis indicate increased intracranial pressure

 

A client with seizure activity is receiving a continuous tube feeding via a small-bore enteral tube. The nurse prepares to administer phenytoin oral suspension via the enteral route. What is the nurse’s priority action before administering this medication?

 

· . Stop the feeding for I to 2 hours (29%)

Phenytoin (Dilantin) is an anticonvulsant drug commonly used to treat seizure disorders. Steady absorption is necessary to maintain a therapeutic dosage range and drug level to control seizure activity. The nurse’s priority action is to stop the feeding for 1 to 2 hours before and after administering phenytoin as products containing calcium (eg, antacidscalcium supplements) and/or nutritional enteral tube feedings can decrease the absorption and the serum level of this drug.

(Option 1) Unless clients have renal insufficiency, renal function tests are not routinely monitored during prescribed phenytoin therapy. Phenytoin is metabolized in the liver and can cause liver damage. Monitoring of liver function test during therapy is recommended.

(Option 2) Flushing the tube with 30-50 ml of water before and after administering phenytoin is recommended to minimize drug loss and drug-drug incompatibility. Flushing with normal saline before and after drug administration is recommended in clients receiving intravenous (IV) phenytoin.

(Option 4) BP is not usually affected in clients prescribed oral phenytoin therapy for seizure disorders. However, IV phenytoin can cause hypotension and arrhythmias.

 

Educational objective:

Phenytoin is an anticonvulsant drug commonly used to treat seizure disorders. Steady absorption is necessary to maintain a therapeutic dosage range and drug level to control seizure activity. Administration of phenytoin concurrent with certain drugs (eg, antacids, calcium) and/or enteral feedings can affect the absorption of phenytoin.

 

The nurse is reviewing new prescriptions for assigned clients. Which prescription would require further clarification from the health care provider?

· Alteplase for an ischemia stroke in a client with a blood pressure of 192/112 mm Hg (l6o/o) 2:

 

 

 

Thrombolytic agents (eg, alteplase, Tenecteplase, reteplase) are often prescribed to resolve acute thrombotic events (eg, ischemie stroke, myocardial infarction, massive pulmonary embolism). They are recombinant plasminogen activators that activate the blood fibrinolytic system and dissolve thrombi.

Thrombolytic agents are contraindicated in clients with active bleeding, recent trauma, aneurysm, arteriovenous malformation, history of hemorrhagic stroke, and uncontrolled hypertension (blood pressure >180/110 mm Hg). Therefore, the health care provider should be consulted for clarification. Administering alteplase in the presence of these conditions can cause hemorrhage, including life-threatening intracerebral hemorrhage (Option 1).

 

(Option 2) Most penicillin derivates (eg, ampicillin, amoxicillin) and cephalosporins (eg, cephalexin, ceftriaxone) are generally considered safe for use by women who are pregnant or lactating.

 

(Option 3) Fentanyl is appropriate in postoperative clients with moderate to severe pain, even those with a history of allergies to codeine. Both drugs have opiate agonist effects but are chemically different. Codeine is a derivative of natural opiates (eg, morphine), whereas fentanyl is completely synthetic.

 

(Option 4) Syndrome of inappropriate antidiuretic hormone (SIAOH) secretion results in water retention and dilutional hyponatremia. Clients with SIADH often require hypertonic saline for sodium repletion to increase serum sodium levels with a minimal infused volume of water.

Educational objective:

Thrombolytic agents (eg, alteplase, Tenecteplase, reteplase) place clients at risk for bleeding. Therefore, they are contraindicated in clients with active bleeding, recent trauma, aneurysm, arteriovenous malformation, history of hemorrhagic stroke, and uncontrolled hypertension.

 

A client diagnosed with trigeminal neuralgia is given a prescription of carbamazepine by the health care provider. Which intervention does the nurse add to this client’s care plan?

 

· Encourage. client to report. any fever or sore throat

 

 

Trigeminal neuralgia is sudden, sharp pain along the distribution of the trigeminal nerve. The symptoms are usually unilateral and primarily in the maxillary and mandibular branches. Clients may experience chronic pain with periods of less severe pain, or “cluster attacks” of pain between long periods without pain. Triggers can include washing the face, chewing food, brushing teeth, yawning, or talking. Pain is severe, intense, burning, or electric shock-like. The primary intervention for trigeminai neuralgia is consistent pain control with medications and lifestyle changes. The drug of choice is carbamazepine. lt is a seizure medication but is highly effective for neuropathic pain. Carbamazepine is associated with agranulocytosis (leukopenia} and infection risk. Clients should be advised to report any fever or sore throat.

Behavioral. interventions include the following:

1. Oral care – use a small soft-bristled toothbrush or a warm mouth wash

2. Use lukewarm water, avoid beverages or food› that are too hot or cold (Option 1)

3. Room shouJ6 be kept at an even and moderate temperature

4. Avoid rubbing or facial massage. Use cotton pads to wash the face if necessary.

5 Have a soft diet with high calorie content; avoid foods that are difficult to chew. Chew on the unaffected I side of the mouth.

 

(Option 2) A high-fiber diet is not required for a client with trigeminai neuralgia, and the additional chewing with higher-fiber foods may serve as a pain trigger.

(Option 3) Clients with trigeminai neuralgia are encouraged not to massage the face as this can trigger pain.

Educational objective:

The primary intervention for trigeminai neuralgia includes pain control and limiting pain triggers. The drug of choice is carbamazepine. Triggers can include washing the face, chewing food, brushing teeth, yawning, or talking. Carbamazepine is associated with agranulocytosis (leukopenia) and infection risk. Clients should be advised to report any fever or sore throat.

 

 

A client having an ischemie stroke arrives at the emergency department. The health care provider prescribes tissue plasminogen activator (tPA). Which client statement would be most important to clarify before administering tPA?

· 1. “I cant believe this is happening right after my stomach surgery.” (42°/o)

 

 

Tissue plasminogen activator (tPA) dissolves clots and restores perfusion in clients with ischemic stroke. lt must be administered within a 3- to 41/2-hour window from onset of symptoms for full effectiveness. The nurse assesses for contraindications to tPA due to the risk of hemorrhage.

The client should not have a history of intracranial hemorrhage or be actively bleeding. Surgery within the last 2 weeks is a contraindication as tPA dissolves all clots in the body and may disrupt the surgical site. This client indicates a recent stomach surgery, which would need further clarification to determine eligibility to receive tPA (Option 1).

 

(Option 2) A client’s history of stroke or head trauma in the last 3 months could exclude tPA use.

 

(Option 3) The nurse should determine when the client first developed stroke symptoms. tPA can be administered if symptoms started within the last 3 to 41/2 hours or based on facility guidelines.

 

(Option 4) Current anticoagulant use may exclude a client from receiving tPA. The duration of action for warfarin is 2-5 days; this client can safely receive tPA as warfarin was discontinued 4 weeks ago. However, if pending coagulation studies drawn prior to tPA administration are elevated, the infusion may be discontinued.

 

Educational objective:

Tissue plasminogen activator (tPA) dissolves clots in an ischemie stroke and must be administered within a 3- to 4Y2-hourwindow from onset of symptoms. The nurse assesses for contraindications to tPA due to the risk of hemorrhage.

 

The nurse in the same-day surgery unit admits a client who will receive general anesthesia. The client has never hadl surgery before. Which question is most critical for the nurse to ask the client dung the preoperative assessment and health history?

· “Has any family member ever had a bad reaction. to general anesthesia?” (59%)

 

 

Malignant hyperthermia (MH) is a rare but life-threatening inherited muscle abnormality that is triggered by specific, inhaled

anesthetic agents and the depolarizing muscle relaxant succinylcholine (Anatine) used to induce general anesthesia.

In MH-susceptible clients, the triggering agent leads to excessive release of calcium from the muscles, leading to sustained muscle

contraction and rigidity (usually of the jaw and upper body [early sign]), increased oxygen demand and metabolism, and dangerously

high temperature (later sign).

As MH is an inherited condition, proper screening and a thorough preoperative nursing assessment and health history can help

minimize the client’s risk (Option 1).

(Option 2) Cervical spine problems should be assessed before the intubation. Low back pain history is not a priority for general

anesthesia.

(Option 3) It would be appropriate to ask about allergies (eg, drugs, latex). However, asking about an anaphylactic reaction to a bee

sting is not the most critical question.

 

(Option 4) History of prior opioid intake may be helpful, but the most important question is to ask about side effects and allergies.

 

Educational objective:

Malignant hyperthermia (MH) is a rare, life-threatening inherited muscular abnormality that is triggered by specific drugs used to

induce general anesthesia. Therefore, it is critical for the perioperative nurse to screen for MH susceptibility by asking if any of the

client’s blood relatives had ever experienced an adverse reaction to general anesthesia, including unexplained death.

 

 

 

 

A client was prescribed phenytoin (100 mg PO 3 times a day) a month ago. Today, the client has a serum phenytoin level of 32

meg/mL (127 mcmol/L). The nurse notifies the health care provider and expects which prescription?

· 2 Decrease phenytoin daily dose (75°/o)

 

 

Phenytoin (Dilantin) is an anticonvulsant drug used to treat generalized tonic-clonic seizures. The therapeutic serum phenytoin range

is 10-20 mcg/mL (40-79 mcmoll/L). In the presence of an elevated phenytoin level (32 mcg/mL [127 mcmol/L]), the nurse anticipates

that the health care provider will prescribe a decreased daily dose (Option 2). The nurse should continue to monitor for signs of

toxicity, typically presenting as neurological manifestations (eg, ataxia, nystagmus, slurred speech, decreased mentation).

(Options 1 and 3) The serum phenytoin level is above the therapeutic level, so administering the prescribed dose or increasing the

dose can further increase the risk for drug-induced toxicity.

(Option 4) Repeating the serum phenytoin level in 2 hours will not result in a significant change because the average half-life of the

drug is 22 hours.

 

Educational objective:

Phenytoin (Dilantin) is used to treat generalized tonic-clonic seizures. Common symptoms of phenytoin toxicity involve the central

nervous system (eg, nystagmus, ataxia, slurred speech, decreased mentation) and can occur when serum phenytoin levels exceed

the therapeutic range (10-20 mcg/mL [40-79 mcmol/L]).

 

Phenytoin (Dilantin) is a widely-used anticonvulsant medication that acts on the central nervous system without depressing the entire CNS. This medication is indicated for partial seizures and tonic-clonic seizures. Side effects of this drug include gingival hyperplasia, sedation, ataxia, nystagmus, rash, purple glove syndrome, anemia, and hirsutism. Since Phenytoin has a narrow therapeutic range and dosing is highly individualized, the patient should strictly adhere to the prescribed dosage and avoid abrupt withdrawal.

 

To remember phenytoin, think of a phone tow truck company with the motto, “You call, we haul.” It shows up to car accidents and clears the road. Butin order to do this, it has to block the road first, leaving all the rush hour salt shakers stuck in the channel. It clears vehicles and any passengers who were ejected during the crash, like tonic clown Caesar here, who was just catching’ a taxi ride when the driver, Nostradamus, had a vision and veered off the road. Apparently, he had been driving while sedated. When witnesses saw this, he got his gummy teeth punched out by a purple glove. The bearded woman was merely returning the favor, as the taxi had hit her and caused a road rash. That, and he ran over her pet anemone. All in a day’s work for a phone tow truck driver, though every crash was different in its own way, requiring personalized dosing to clear the scene.

The nurse is caring for a client who is taking riluzole for amyotrophic lateral sclerosis (ALS). The client asks, “There’s no cure for ALS,

so why should | keep taking this expensive drug?” What is the nurse’s best response?

· 1: “It may be able to slow the progression of ALS.” (7%)

 

 

 

 

Amyotrophic lateral sclerosis (ALS), also known as Lou Gehrig disease, is a debilitating, progressive neurodegenerative disease

with no cure. Clients develop fatigue and muscle weakness that progresses to paralysis, dysphagia, difficulty speaking, and

respiratory failure. Most clients diagnosed with ALS survive only 3-5 years.

Riluzole (Rilutek) is the only medication approved for ALS treatment. Riluzole, a glutamate antagonist, is thought to slow neuron

degeneration by decreasing the production and activity of the neurotransmitter glutamate in the brain and spinal cord. In some clients,

riluzole may slow disease progression and prolong survival by 3-6 months. The nurse should provide teaching about the purpose of

the medication so that the client can make an informed decision about taking it (Option 1).

(Option 2) Explaining the pharmacology of riluzole is not the best response for helping the client understand the purpose of taking the

medication.

(Option 3) It would be appropriate to consult the case manager if the client expresses concern about not having the appropriate

resources to acquire a costly medication, but the nurse should first ensure that the client understands the medication’s purpose.

 

(Option 4) The client has the right to refuse any medication, but the nurse should first ensure that the client is informed and

understands the purpose of the medication.

Educational objective:

Although there is no cure for amyotrophic lateral sclerosis, the medication riluzole may slow disease progression and prolong survival.

 

Amyotrophic Lateral Sclerosis or ALS, can be remembered by Lou Gehrig holding an emmy-trophy ladder on a bed of skull-roses. This rare neurologic disorder is characterized by progressive muscle weakness, shown by the progressively weaker muscle, due to degeneration of motor neurons in the brainstem and spinal cord. It results in systemic muscle wasting, shown by the systemic muscle waste-basket, usually in the limbs with atrophy of muscles in the hands as the first sign. Other symptoms include fasciculations, shown by the fast-pickle, which is twitching and cramping of muscles, especially those in the hands and feet. Patients will also display spasticity, shown by the spaz-tick, which is stiff or rigid muscles, along with fatigue, shown by the sleepy-guy. There is no cure for ALS however, certain interventions can help with symptoms. Currently, the drug Riluzole (Rilutek), shown by the reel-u, has been approved to help slaw the progression of the disease by decreasing the amount of glutamate in the brain. Additionally, encouraging moderate intensity exercise, shown by the moderate exercise- guy, can help reduce spasticity for the trunk and limbs. It is important to understand there is no cognitive decline in ALS, shown by the down- arrow brain no-sign, which means that patients will usually have full cognition until their death. Because this disease affects the muscles that control speech, swallowing, and breathing, patients may develop dysarthria and dysphagia and will eventually need respiratory support, shown by the lungs being supported. This disease is variable but the average life expectancy of an ALS patient is approximately two to five years from the time of diagnosis, therefore patients should be provided with palliative care, shown by the palliative-pail, to promote comfort and improve quality of life.

 

So to summarize, amyotrophic lateral sclerosis (ALS), sometimes referred to as “Lou Gehrig’s Disease” is a rare neurologic disorder

characterized by degeneration of motor neurons in the brainstem and spinal cord leading to progressive muscle weakness and systemic

muscle wasting. Patients may experience fasciculations, spasticity, and fatigue. Interventions include administration of Riluzole, promoting moderate exercise, and being aware that there is no cognitive decline in this disease. Patients will likely need respiratory support and will often require palliative care at the end of life.

Lou Gehrig earns himself a Emmy-trophy Ladder on Skull-roses but, all he wants to do is play baseball. However, he can’t because the field is full with inept players that ruin his game. It all started when a wild pitch flies towards the stands and the first baseman attempts to catch it but can’t due to his Progressively Weaker Muscle. Which isn’t that weird, being that first baseman is a Systemic Muscle waste-basket. Anyway the Fast-pickle sees this as his chance to run a pickle and he races towards the next base. Wow just imagine, none of this would happen if it weren’t for that Spaz-tick pitcher that can’t throw a ball. Now Sleepy-guy tries to slide into home, but of course he falls asleep in the process. Over in the dugout a team mate, Moderate Exercise-guy attempts to lure sleeping guy to home base by Reeling a U but fails. Another player isn’t in such great of shape. It’s the Lungs who need support, even though he can hardly move or talk he shows No-signs of Cognitive Decline. His pal, Palliative-pail, helps ease his pain by caring for him.

 

 

 

 

The health care provider (HCP) has prescribed amitriptyline 25 mg orally every morning for an elderly client with recent herpes zoster

infection (shingles) and severe postherpetic neuralgia. What is the priority nursing action?

· Teach the client to get up slowly from the bed or a sitting position Tricyclic antidepressants (eg, amitriptyline, nortriptyline, desipramine, imipramine) are commonly used for neuropathic pain. Side effects are especially common in elderly clients.

 

Side effects & nursing interventions for clients taking tricyclic antidepressants

 

 

 

 

Cardiovascular

· Dizziness, orthostatic hypotension increased risk of falling, tachycardia

· Teach client to change positions sIowIy, avoid hot baths/showers, and avoid alcohol and! Sedatives

· Obtain baseline electrocardiogram on elderly clients

· Monitor blood pressure and pulse

 

 

 

 

Anticholinergic

· Dry mouth, constipation urinary retentions and/or difficulty initiating a urinary stream, blurred vision

· Teach clients to drink sips of water, sick on ice chips, chew sugarless gum

· Teach client to consume a highs-fiber diet and large

amounts of fluids

· Contact HCP if urinary problems persist or cause pain

· Report changes in vision to the HCP; reassure client

that. blurred vision is temporary

 

 

 

Neurological

· Drowsiness, confusion

· Teach client to use caution when driving or engaging in other activities requiring close concentration

· Ask HCP if medication can be administered prior to bedtime rather than in morning

 

 

 

 

 

 

Neurological I

 

· Drowsiness. Confusion

· Teach client to use caution when driving or engaging in other activities requiring close concentration

· Ask HCP if medication can be administered prior to bedtime rather than in mornings

 

Dermatological

· Photosensitivity

· Teach client to wear sunscreen„ sunglasses, and protective clothing when outdoors

 

 

 

 

Due to the increased risk of falling, the priority nursing action is to teach the client to get up slowly from the bed or a sitting position.

(Options 1, 2, and 4) These are important instructions but not priority ones.

Educational objective:

The most common side effects experienced by clients taking tricyclic antidepressants include dizziness, drowsiness, dry mouth,

constipation, photosensitivity, urinary retention, and blurred vision. The priority nursing action is to teach caution in changing positions

due to the increased risk for falls from dizziness and orthostatic hypotension, especially in elderly clients.

 

Tricyclic antidepressants or TCAs, shown by the tricycle ant-tie-depressed-emo, are a class of medications that work by inhibiting the

reuptake of norepinephrine and serotonin, shown by the inhibiting-chains on the re-uptake tube with north-epi-pen and silver-tonic. TCAs are primarily indicated to treat moderate to severe depression, illustrated by the depressed-emo, but they can also be helpful in treating depressive episodes of a patient with bipolar disorder, shown by the bi-polar-bear. The TCA amitriptyline can be used to treat fibromyalgia, shown by Fabio-mayo-algae. Side effects of TCAs include sedation, shown by the sedation-darts. Patients may also experience orthostatic hypotension, the oar hippo-BP, along with anticholinergic effects, depicted as the ant-tie-cola. Anticholinergic effects include constipation, urinary retention, dry mouth, photophobia, blurred vision, and tachycardia. The most serious adverse effect of TCAs is cardiac toxicity, illustrated as the heart with toxic-green-glow. Due to their side effects, TCAs have largely been replaced by safer and better tolerated alternatives. An important teaching point is to remind patients that TCAs may require 2 weeks or longer for therapeutic effects to occur. This delayed effect is represented by the delayed-sign. Lastly, individuals taking a TCA should not abruptly stop their medication, shown by the can’t stop cold-turkey. Instead, they should be tapered off to minimize discontinuation syndromes and cholinergic rebound effects.

 

So let’s review tricyclic antidepressants or TCAs. These drugs work by inhibiting the reuptake of norepinephrine and serotonin for treatment of depression, bipolar disorder, and fibromyalgia. Side effects of TCAs include sedation, orthostatic hypotension, anticholinergic effects, and cardiac toxicity. Due to their side effects, TCAs have largely been replaced by safer and better tolerated alternatives. These medications have a delayed onset of effect, and patients should be instructed to not abruptly stop taking this medication.

 

The tricycle ant-tie-depressed-emo’s are like the depression police, and they patrol the highways on their tricycles, looking for any emos

drinking silver-tonic while driving or shooting up north-epi-pens. They wrap inhibiting-chains this is when the ant-ties confiscate the silver-tonic and north-epi-pens, which start to pile up. They hand cuff all the emos, and wrestle his partner in crime, Fabio, to the ground for abusing mayo-algae, causing extensive muscle pain. Bi-polar-bears are more resistant to arrest and flee, causing the ant-ties to fire a

barrage of sedation-darts to stop it. This causes some unwanted side effects, as the hail of darts end up hitting innocent bystanders, like the oar hippo-BP steering a gondola with two lovers. An ant-tie was just about to pour some cola for his date, the heart, when he was hit by the darts, causing the heart to mournfully scream and become toxic, having lost her lover. Also, something to consider is that these highway roadblocks caused major traffic delays. And try as they might to screen everyone for these illicit substances, the ant-tie emos just can’t stop a cold-turkey.

 

 

 

The home health nurse prepares to give benztropine to a 70-year-old client with Parkinson disease. Which client statement is most

concerning and would warrant health care provider notification?

· “I am going for repeat testing to confirm glaucoma.”

 

Parkinson disease (PD) is a progressive neurological disorder characterized by bradykinesia (loss of autonomic movements), rigidity,

and tremors. Clients with PD have an imbalance between dopamine and acetylcholine in which dopamine is not produced in high

enough quantities to inhibit acetylcholine. Anticholinergic medications (eg, benztropine, trihexyphenidyl) are commonly used to

treat tremor in these clients. However, in clients with benign prostatic hyperplasia or glaucoma, caution must be taken as anticholinergic drugs can precipitate urinary retention and an acute glaucoma episode. As a result, such medications are

contraindicated in these clients.

 

(Option 2) Decreased ability to exercise is common in clients with PD due to tremors and bradykinesia, and they require physical and

occupational therapy consultations. However, acute glaucoma can be sight threatening and is the priority.

 

(Option 3) Esomeprazole is safe to take with benztropine and will not cause an adverse reaction.

 

(Option 4) Constipation is a common side effect of benztropine. Due to the characteristic decreased mobility, PD can also cause

constipation. The client should be instructed to increase dietary fiber intake and drink plenty of water. However, this is not the most

concerning issue.

Educational objective:

Anticholinergic medications (eg, benztropine, trihexyphenidyl) are used to treat Parkinson disease tremor. However, they can

precipitate acute glaucoma and urinary retention and are therefore contraindicated in susceptible clients (eg, those with glaucoma or

benign prostatic hyperplasia).

 

Anticholinergics, personified by the ant-tie-cola, are muscarinic antagonists. These drugs can be remembered through the “B.O.AT.S.”

acronym, hence the boats in the image. The first drug is benztropine, who is personified by the Benz-trooper. This drug can be used as a

second line agent to treat Parkinson’s disease, Park-in-sun parking garage. Oxybutynin, the ox-beauty, works to decrease bladder spasms, represented by the down-arrow bladder spaceship. A drug with a wide range of uses is atropine, the @-trooper, and it causes mydriasis and cycloplegia, depicted with the meter-eyes and paralyzed eye. Scopolamine, shown by the scope-lamb, is used to treat motion sickness by acting on the vestibular system, depicted through the seasick character.

 

So don’t forget! Anticholinergic drugs can be remembered through the B.O.AT.S. acronym. Parkinson’s is treated by benztropine, which you can remember with the phrase “Park my Benz.” Oxybutynin decreases bladder spasms and atropine causes mydriasis and cycloplegia.

Scopolamine is used for motion sickness.

Summer’s here, and the ant-tie-colas are out to party. They guzzle down colas and get reckless with their B.O.AT.S. The park-in-sun parking garage is not the place for parking boats, so the Benz-troopers will be patrolling the park-in-sun, on the lookout for illegal parking. Sunshine means it’s time to show off your beach body. But don’t let yourself be distracted by Ox-beauties on the beach, otherwise you’ll end up like the bladder spaceship and come crashing down. The @-troopers are out in full force, looking for anyone who might try taking advantage of those always empty no handicap parking spots. The paralyzed eye gets more than a warning for illegally parking his wheelchair, and as you can tell by his meter-eye, he wishes he had just paid the meter. Inexperienced boaters thank their lucky stars that scope-lamb is here to help. She snipes the engine of any ant-tie-colas who lose control and get seasick. Of course, they’re left floating out to sea, but at least they’re not sick. So, remember ant-tie’s, don’t drink cola and drive boats. This message brought you by your local state troopers. Please drink responsibly.

 

Remember it like this; Ant-tie-colas like to party on B.O.AT.S. When they illegally park in the park-in-sun garage, the Benz-trooper hands out tickets. Then the ox-beauty steps onto the beach, catching everyone’s attention. And the bladder spaceship comes crashing down when distracted. The @-trooper tickets the meter-eye in a wheelchair for parking in a no handicap spot and paying the meter. And a scope-lamb snipes the engine of an out-of-control anti-cola with sea sickness

 

The nurse reviews the analgesia prescriptions for assigned clients. The nurse should question the health care provider about which

prescription?

· Transdermal fentanyl patch for a client who is 1 day postoperative above-the-knee amputation and reports intermittent, throbbing stump pain (26%)

 

transdermal fentanyl patch is prescribed for clients suffering from moderate to severe chronic pain. The patch provides

continuous analgesia for up to 72 hours. However, the drug is absorbed slowly through the skin into the systemic circulation and can

take up to 17 hours to reach its full analgesic effect. Therefore, it is not recommended for treating acute postoperative, temporary, or

intermittent pain as it does not provide immediate analgesia when applied.

 

(Option 1) A lidocaine 5% transdermal patch provides a localized, topical anesthetic to intact skin. It is commonly prescribed for

clients with chronic postherpetic neuralgia, a painful, debilitating condition that can develop following a herpes zoster (shingles)

infection.

 

(Option 2) The client with opioid abuse history would be experiencing the same type and degree of pain as other clients with a

fractured femur. However, a higher dose or a stronger opioid analgesic (eg, hydromorphone) is needed for pain relief due to the

client’s increased opioid tolerance.

 

(Option 3) Tramadol is a synthetic opioid analgesic prescribed to treat moderate to severe postoperative pain. It is appropriate to

prescribe at discharge as it has fewer complications related to respiratory depression compared with other opioids.

 

Educational objective:

A transdermal fentanyl patch is indicated to treat moderate to severe chronic pain. It is not recommended for treating acute

postoperative, temporary, or intermittent pain as it does not provide immediate analgesia when applied.

 

 

 

The home health nurse reviews the serum laboratory test results for a client with seizures. The phenytoin level is 27 mcg/mL. The

client makes which statement that may indicate the presence of dose-related drug toxicity and prompt the nurse to notify the health

care provider?

· “I am. feeling unsteady when I walk.”

 

Phenytoin (Dilantin) is an anticonvulsant drug used to treat generalized tonic-colonic seizures. The therapeutic serum phenytoin

reference range is between 10-20 mcg/mL. Levels are measured when therapy is initiated, periodically throughout treatment to guide

dosing until a steady state is attained (3-12 months), and if seizure activity increases. Early signs of toxicity include horizontal nystagmus and gait unsteadiness. These may be followed by slurred speech, lethargy, confusion, and even coma. Bradyarrhythmia’s and hypotension are usually seen with intravenous phenytoin.

 

(Option 2) Nocturia is an expected side effect of diuretics but not phenytoin. Nocturia is also seen with diabetes mellitus and benign prostatic hyperplasia.

(Option 3) Metallic taste in the mouth is often seen with metronidazole but not with phenytoin.

 

(Option 4) Gingival hyperplasia is a common expected side effect of phenytoin and does not indicate drug toxicity. It occurs more

often in clients <23 years of age who are prescribed >500 mg/day. Good oral hygiene can limit symptoms.

 

Educational objective:

Phenytoin, an anticonvulsant drug, is used to treat generalized tonic-clonic seizures. Common symptoms of phenytoin drug-induced

toxicity involve the central nervous system and include ataxia, nystagmus, slurred speech, and decreased alertness.

 

To remember phenytoin, also known by its trade name Dilantin, think of a phone tow truck company with the motto, “You call, we haul.” It shows up to car accidents and clears the road, but in order to do this, it has to block the road first, leaving all the rush hour salt shakers stuck in the channel. This represents that phenytoin works by blocking Na (sodium) channels. It is indicated for use in tonic-clonic seizures, shown by the tonic-clown Caesar who was standing around trying to catch A-taxi, representing the possible side effect of ataxia, or uncoordinated movements. Patients taking phenytoin may also experience a condition of involuntary eye movements known as nystagmus, which is shown by the driver Nostradamus who is the one responsible for this wreck. Evidently, he had been driving with a sedation-dart, representing that patients may also experience sedation while taking this medication, especially with high doses. Gingival hyperplasia is another side effect, shown by the gums and teeth wind-up toy that falls to the ground as Caesar gets punched in the face by purple gloves, signifying the possibility of purple glove syndrome. This is a rare complication of intravenous phenytoin use resulting in largely swollen, discolored, and painful hands and arms. Other possible side effects include hirsutism, shown by the bearded-lady starting this fight, and a rash, the rash on her arm. Anemia may also occur, shown by the pet anemone that got run over in this accident. Lastly, with phenytoin, it is important to monitor therapeutic levels as individualized dosing is necessary, shown by the personalized dosing needle helping to clear the scene.

 

So in quick review. Phenytoin works by blocking Na channels for use in tonic-clonic seizures. Side effects to be aware of include ataxia,

nystagmus, sedation, gingival hyperplasia, purple glove syndrome, hirsutism, rash, and anemia. Considerations include the need for individualized dosing.

 

 

 

The clinic nurse prepares to administer a newly prescribed dose of sumatriptan to a client with a migraine headache. Which item in

the client’s history would cause the nurse to question the prescription?

 

 

· Past medical history of uncontrolled I hypertension (61°/o)

 

 

Sumatriptan is a selective serotonin agonist prescribed to treat migraine headaches, which are thought to be caused by dilated

cranial blood vessels. Triptan drugs, like sumatriptan, work by constricting cranial blood vessels, and clients should be instructed

to take a dose at the first sign of a migraine to help prevent and relieve symptoms. Sumatriptan is contraindicated in clients with coronary artery disease and uncontrolled hypertension because its vasoconstrictive properties increase the risk of angina, hypertensive urgency, decreased cardiac perfusion, and acute myocardial infarction. The nurse should question the client about a past medical history of uncontrolled hypertension and report this to the health care provider (Option 3).

 

(Option 1) A blood urea nitrogen level of 12 mg/dL (4.28 mmol/L) is a normal value (normal range 6-20 mg/dL [2.1-7.1 mmol/L]).

(Option 2) Sumatriptan is not contraindicated for underweight or overweight clients.

 

(Option 4) Sumatriptan is not contraindicated with alprazolam therapy. However, because of its serotonergic effects, clients already

taking selective serotonin reuptake inhibitors (eg, sertraline, paroxetine) or selective norepinephrine reuptake inhibitors (eg,

venlafaxine, duloxetine) should be monitored for signs of serotonin syndrome.

 

Educational objective:

 

Sumatriptan relieves migraines by constricting dilated cranial blood vessels. Sumatriptan is contraindicated in clients with coronary

artery disease and uncontrolled hypertension because the vasoconstrictive effects can cause hypertensive urgency, angina,

decreased cardiac perfusion, and acute myocardial infarction.

So, in summary, remember that sumatriptan, with its rapid-onset and short half-life, is a 5-HT agonist that inhibits trigeminal nerve activity and induces vasoconstriction in the treatment of migraine and cluster headaches. Side effects include an increased risk of serotonin syndrome, potential for paresthesia’s and rarely, coronary vasospasm, which contraindicates its use in patients with a history of Prinzmetal’s angina or severe coronary artery disease.

 

During shift report it was noted that the off-going nurse had given the client a PRN dose of morphine 2 mg every 2 hours for incisional

pain. What current client assessment would most likely affect the oncoming nurse’s decision to discontinue the administration every

2 hours?

 

· Client’s respiratory rates 11/m

 

 

Morphine is an opioid analgesic that can be given intravenously for moderate to severe pain. An adverse reaction to morphine

administration is respiratory depression. A respiratory rate <12/min would be a reason to hold morphine administration. The

nurse should perform a more in-depth assessment of the client’s pain and causes. The morphine dose may need to be decreased or

the time between administrations may need to be increased. The nurse should not administer additional doses until the respiratory

rate increases.

 

(Option 1) Morphine can cause burning during IV administration. This can be reduced by diluting the morphine with normal saline

and administering it slowly over 4-5 minutes.

 

(Option 2) The nurse should instruct the client to call for help before getting up to go to use the bathroom to avoid falls caused by

dizziness from the morphine.

 

(Option 3) Morphine can lower blood pressure, and clients receiving it should have blood pressure monitored. This blood pressure

reading is not severely low and is not a priority over the respiratory depression.

 

Educational objective:

Morphine administration can cause respiratory depression. The nurse should hold a dose of morphine for a client whose respiratory

rate is <12/min.

 

The office nurse, while reviewing a client’s health information, notices that the client has recently started taking St. John’s wort for

symptoms of depression. What additional information is most important for the nurse to obtain?

· Ask. if the cIien.t s currently taking any prescription antidepressant medications (82o/o)

 

 

St. John’s wort is an herbal product commonly used by many clients to treat depression. However, it may interact with medications

used to treat depression or other mood disorders, including tricyclic antidepressants, selective serotonin and/or norepinephrine

receptor inhibitors (SSRIs/SNRIs), and monoamine oxidase inhibitors (MAOIs). Taking St. John’s wort with these medications tends to

increase side effects and could potentially lead to a dangerous condition called serotonin syndrome.

 

Serotonin is a chemical produced by the body that is needed for the nerve cells and brain to function. Excessive serotonin causes

symptoms that can range from mild (shivering and diarrhea) to severe (muscle rigidity, fever, and seizures). Severe serotonin

syndrome can be fatal if it is not treated.

 

(Option 2) The nurse can ask the client if a diagnosis of depression has been made by an HCP, but inquiring about possible

medications that can interact with St. John’s wort is more important at this time.

 

(Option 3) St. John’s wort may interfere with the absorption of iron and other minerals. This is a teaching point, but it is not the

highest priority question to ask the client.

 

(Option 4) St. John’s wort can cause photosensitivity which could be exacerbated by use of tanning beds. However, this is not the

highest priority question to ask the client.

 

Educational objective:

St. John’s wort interferes with many prescription medications. It is a priority for the nurse to assess for concomitant use of St. John’s

wort with prescription SSRIs, MAOIs, or tricyclic antidepressants as such combinations can cause serotonin syndrome.

 

St. John’s wort

A plant product (Hypericum perforatum), not regulated by

the U.S. Food and Drug Administration, is taken by some

individuals to relieve manifestations of mild depression.

NURSING ACTIONS

Adverse effects include photosensitivity, skin rash,

rapid heart rate, gastrointestinal distress, and

abdominal pain.

St. John’s wort can increase or reduce levels of some

medications if taken concurrently. The client should

inform the provider if taking St. John’s wort.

! Medication interactions: Potentially fatal serotonin

syndrome can result if St. John’s wort is taken

with SSRIs or other types of antidepressants.

Foods containing tyramine should be avoided.

Serotonin syndrome

Can begin 2 to 72 hr after starting treatment and can

be lethal.

MANIFESTATIONS

Confusion, agitation, poor concentration, hostility

Disorientation, hallucinations, delirium

Seizures leading to status epilepticus

Tachycardia leading to cardiovascular shock

Labile blood pressure

Diaphoresis

Fever leading to hyperpyrexia

Incoordination, hyperreflexia

Nausea, vomiting, diarrhea, abdominal pain

Coma leading to apnea (and death in severe cases)

CLIENT EDUCATION: Observe for manifestations. If any

occur, withhold the medication and notify the provider.

 

The nurse is reinforcing education to a client newly prescribed levetiracetam for seizures. Which statement made by the client

indicates a need for further instruction?

· “I can begin driving again after I have been on this medication for a few weeks.”

 

 

Levetiracetam (Keppra) is an anticonvulsant prescribed for seizure disorders. As with other antiseizure medications, levetiracetam

has a depressing effect on the central nervous system (CNS), which may cause drowsiness, somnolence, and fatigue as clients

adjust to the medication. Clients should be assured that this is common and typically improves within 4-6 weeks (Option 1).

However, the CNS-depressing effects of levetiracetam may be enhanced if taken with other CNS-depressing substances (eg, alcohol)

or medications.

New or increased agitation, anxiety, and/or depression or mood changes should be reported immediately as levetiracetam is

associated with suicidal ideation (Option 3).

 

Like other anticonvulsants, levetiracetam can trigger Stevens-Johnson syndrome, a rare but life-threatening blistering reaction of the

skin. Rash, blistering, muscle/joint pain, or conjunctivitis should be reported and assessed immediately (Option 4).

 

(Option 2) Clients with seizure disorders should avoid driving or operating heavy machinery until they have permission from their

health care provider and have met the requirements of their department of transportation. Typically, the client must be free from

seizures for an allotted time period.

 

Educational objective:

Levetiracetam is an anticonvulsant prescribed for seizure disorders. It may have depressing effects on the central nervous system

(eg, drowsiness) as the body adjusts to therapy. Serious adverse effects include suicidal ideation and Stevens-Johnson syndrome.

Clients with seizure disorders must meet the guidelines of their department of transportation and receive permission from their health

care provider prior to legally operating a motor vehicle.

 

Cranial nerve assessment.
 

Cranial nerve name

& number

 

Mnemonic.

Sensory (S) Motor (M) Botha (8)  

Assessment

Olfactory (I) On S—Some test
Optic (li) Old S—Say Visual acuity & visuals fields
 

Oculomotor. (Ili)

 

Olympus

 

M/-— Marry

Pupil constriction & extraocular movements
Trochlear IV)  

Towering

 

M—money

Extraocular movements

— inferior adduction

Trigeminal (V) Tops B—But Clench teeth & I/light touch
 

Abducens (VI)

A M—My Extraocular movements

— lateral/ abduction

 

Facial (VII)

Finn B-brother Facial/ movement— close eyes, smile
     
Acoustic (VII I›) And S–Ways Hearing & Romberg test
Glo6sopharyngeal (IX) German B—Bad. Gag reflex
 

Vigus(X)

 

Viewed

 

B — Business

Say ‘Kahn’ – uvular and I palate movement
 

Spinal accessory (XI)›

 

Some

 

ML- Marry

Turn› heads & lift shoulders to resistance
Hypoglossal (XII) Hops MI—Money Stick. out tongue